+ All Categories
Home > Health & Medicine > MedicalResearch.com: Medical Research Exclusive Interviews November 26 2014

MedicalResearch.com: Medical Research Exclusive Interviews November 26 2014

Date post: 11-Jul-2015
Category:
Upload: marie-benz-md-faad
View: 416 times
Download: 0 times
Share this document with a friend
96
MedicalResearch.com Exclusive Interviews with Medical Research and Health Care Researchers from Major and Specialty Medical Research Journals and Meetings Editor: Marie Benz, MD [email protected] November 26 2014 For Informational Purposes Only: Not for Specific Medical Advice.
Transcript
Page 1: MedicalResearch.com:  Medical Research Exclusive Interviews November 26 2014

MedicalResearch.comExclusive Interviews with Medical Research and

Health Care Researchers from Major and Specialty Medical Research Journals and Meetings

Editor: Marie Benz, MD [email protected]

November 26 2014

For Informational Purposes Only: Not for Specific Medical Advice.

Page 2: MedicalResearch.com:  Medical Research Exclusive Interviews November 26 2014

Medical Disclaimer | Terms and Conditions

• The contents of the MedicalResearch.com Site, such as text, graphics, images, and other material contained on the Hemodialysis.com Site ("Content") are for informational purposes only. The Content is not intended to be a substitute for professional medical advice, diagnosis, or treatment. Always seek the advice of your physician or other qualified health provider with any questions you may have regarding a medical condition. Never disregard professional medical advice or delay in seeking it because of something you have read on the Hemodialysis.com Site!

• If you think you may have a medical emergency, call your doctor or 911 immediately. MedicalResearch.com does not recommend or endorse any specific tests, physicians, products, procedures, opinions, or other information that may be mentioned on the Site. Reliance on any information provided by MedicalResearch.com or other Eminent Domains Inc (EDI) websites, EDI employees, others appearing on the Site at the invitation of MedicalResearch.comor EDI, or other visitors to the Site is solely at your own risk.

• The Site may contain health- or medical-related materials that are sexually explicit. If you find these materials offensive, you may not want to use our Site. The Site and the Content are provided on an "as is" basis.

Read more interviews on MedicalResearch.com

Page 3: MedicalResearch.com:  Medical Research Exclusive Interviews November 26 2014

Smoking Cessation Key To Improving Quality of Life in Asthmatic AdolescentsMedicalResearch.com Interview with:

Dr. Wanjun Cui, MS PhDDepartment of Health and Human Services,

Centers for Disease Control and Prevention (CDC)

• Medical Research: What is the background for this study? What are the main findings?

• Authors’ response: Asthma is a leading chronic disease among adolescents that adversely affects their health. However, it is unclear how asthma influences their perceived health or health-related quality of life (HRQOL). Because their perceptions of their health may differ from those of their caregivers (such as parents or health professionals), knowing how adolescents with asthma would rate their own health is very important. Our study compares the responses of adolescents with and without asthma about different aspects of HRQOL including their overall health, their recent physical health, their recent mental health, and their recent activity limitations due to health. Unlike previous U.S. studies based on small clinical samples, our study used a nationally representative sample of U.S. adolescents that can be generalized to the whole U.S. adolescent population.

• We found that asthma is adversely associated with almost all these aspects of HRQOL but only among those with asthma and current symptoms such as wheezing and dry cough. Adolescents with asthma without current symptoms did not report significantly worse HRQOL than those without asthma. For example, compared with those who never had asthma, adolescents with asthma and symptoms of dry cough or wheezing reported significantly more fair or poor self-rated health (14% vs. 8%), 34% more recent physically unhealthy days , and 26% more recent mentally unhealthy days. More importantly, adolescents with asthma who currently smoked cigarettes or reported limited physical functioning reported even worse physical and mental HRQOL.

Read the rest of the interview on MedicalResearch.comContent Not Intended as Specific Medical Advice

Page 4: MedicalResearch.com:  Medical Research Exclusive Interviews November 26 2014

Smoking Cessation Key To Improving Quality of Life in Asthmatic AdolescentsMedicalResearch.com Interview with:

Dr. Wanjun Cui, MS PhDDepartment of Health and Human Services,

Centers for Disease Control and Prevention (CDC)

• Medical Research: What should clinicians and patients take away from your report?

• Authors’ response: Clinicians and other health professionals should help adolescents with asthma to control their asthma symptoms to improve the adolescents’ HRQOL. Since the 15% of adolescents with asthma and symptoms who currently smoked cigarettes reported even worse HRQOL, parents and clinicians should encourage these adolescents to quit smoking to improve their mental and physical health and HRQOL. Finally, because about 20% of adolescents with asthma and symptoms reported limited physical functioning and worse HRQOL, clinicians and other health professionals should both control these adolescents’ asthma symptoms and improve their physical functioning to improve their mental and physical HRQOL.

Read the rest of the interview on MedicalResearch.comContent Not Intended as Specific Medical Advice

Page 5: MedicalResearch.com:  Medical Research Exclusive Interviews November 26 2014

Smoking Cessation Key To Improving Quality of Life in Asthmatic AdolescentsMedicalResearch.com Interview with:

Dr. Wanjun Cui, MS PhDDepartment of Health and Human Services,

Centers for Disease Control and Prevention (CDC)

• Medical Research: What recommendations do you have for future research as a result of this study?

• Authors’ response: Although our research has shown adolescents have better HRQOL than adults, many adolescents still engage in risky behaviors such as smoking and drinking alcohol often associated with worse HRQOL. We thus recommend that future research should consider these behaviors when examining HRQOL in adolescents with health conditions because these behaviors might affect associations between these conditions and HRQOL.

• Citation:

• Health-Related Quality of Life and Asthma among United States Adolescents

• Cui, Wanjun et al.The Journal of Pediatrics Published Online: November 13, 2014

• DOI: http://dx.doi.org/10.1016/j.jpeds.2014.10.005

Read the rest of the interview on MedicalResearch.comContent Not Intended as Specific Medical Advice

Page 6: MedicalResearch.com:  Medical Research Exclusive Interviews November 26 2014

New Cholesterol Management Guidelines Not Fully Implemented in Medical PracticeMedicalResearch.com Interview with:

Thomas M. Maddox MD MScCardiology, VA Eastern Colorado Health Care System Associate Director, VA CART Program

Associate Professor, Department of Medicine University of Colorado School of Medicine

• Medical Research: What is the background for this study? What are the main findings?

• Dr. Maddox: With the release of the updated cholesterol guidelines last year and their significant changes in recommendations, we wanted to see what the potential impact would be on U.S. cardiovascular practices. Specifically, we were interested in present cholesterol treatment and testing patterns, and how they would potentially need to change under the new guidelines.

• We used the PINNACLE registry to conduct our investigation. Under the sponsorship of the American College of Cardiology, the registry collects EMR data from 111 cardiovascular practices around the U.S. We analyzed cholesterol treatment and testing patterns in approximately 1.2 million patients. We found that most patients qualified for cholesterol treatment with statins, but 32.4% weren’t currently prescribed them. We also found that 22.6% of patients were being treated with non-statin lipid-lowering therapies which, under the new guidelines, aren’t currently recommended for cholesterol treatment. Finally, we found that 20.8% of patients underwent repeated LDL-C testing, which may not be necessary under the new guidelines.

• Medical Research: What should clinicians and patients take away from your report?

• Dr. Maddox: Clinicians should examine their own practice, and ensure that the cholesterolmanagement of their patients hews to the latest evidence. Doing so will maximally reduce our patients’ collective risk for myocardial infarction and death.

Read the rest of the interview on MedicalResearch.comContent Not Intended as Specific Medical Advice

Page 7: MedicalResearch.com:  Medical Research Exclusive Interviews November 26 2014

New Cholesterol Management Guidelines Not Fully Implemented in Medical PracticeMedicalResearch.com Interview with:

Thomas M. Maddox MD MScCardiology, VA Eastern Colorado Health Care System Associate Director, VA CART Program

Associate Professor, Department of Medicine University of Colorado School of Medicine

• Medical Research: What recommendations do you have for future research as a result of this study?

• Dr. Maddox: Quality improvement initiatives are needed to examine the reasons behind these gaps in treatment and, where possible, close them. The recent release of the IMPROVE-IT trial, which demonstrated that ezetimibe may have a role in cholesterol management, needs to be incorporated into our evidence. In addition, results of ongoing research in newer cholesterol agents, such as PCSK9 inhibitors and anti-inflammatories, will need incorporation, where relevant, into our management decisions.

• Citation:

• Maddox TM, Borden WB, Tang F, et al. Implications of the 2013 ACC/AHA Cholesterol Guidelines for Adults in Contemporary Cardiovascular Practice: Insights From the NCDR PINNACLE Registry. J Am Coll Cardiol. 2014;():. doi:10.1016/j.jacc.2014.08.041.

Read the rest of the interview on MedicalResearch.comContent Not Intended as Specific Medical Advice

Page 8: MedicalResearch.com:  Medical Research Exclusive Interviews November 26 2014

Maternal Touch Activates Genes That Alleviate Pain in InfantsMedicalResearch.com Interview with:

Regina Marie Sullivan PhD Professor Child and Adolescent Psychiatry Nathan Kline InstituteThe Child Study Center at NYU Langone Medical Center

Department of Child and Adolescent Psychiatry One Park Ave 8th Floor, New York, NY 10016

• Medical Research: What is the background for this study?

• Dr. Sullivan: Managing pain during medical procedures in a critically important issue in medicine today. Our study was designed to better understand one method of reducing pain in young infants – having the caregiver be in contact with the baby during the painful procedure, which reduces the infant’s behavioral response to the medical procedure. This study explored the neural basis of the ability of the caregiver to reduce the pain response.

Medical Research: What are the main findings?

• Dr. Sullivan: This research has shown us is that the maternal presence during pain is not just dampening the brains response to pain – this procedure is activating a unique set of genes involved in brain development.

Read the rest of the interview on MedicalResearch.comContent Not Intended as Specific Medical Advice

Page 9: MedicalResearch.com:  Medical Research Exclusive Interviews November 26 2014

Maternal Touch Activates Genes That Alleviate Pain in InfantsMedicalResearch.com Interview with:

Regina Marie Sullivan PhD Professor Child and Adolescent Psychiatry Nathan Kline InstituteThe Child Study Center at NYU Langone Medical Center

Department of Child and Adolescent Psychiatry One Park Ave 8th Floor, New York, NY 10016

• Medical Research: What should clinicians and patients take away from your report?

• Dr. Sullivan: Controlling pain in infants during medical procedures is very important and it helpful to have simple naturally available means to help the infant. It is also very important that we understand both the immediate and long-term costs and benefits of this procedure. In this case, we now understand that this pain reduction procedure is activating a unique set of genes involved in brain development and not simply attenuating the brain’s pain response.

• Medical Research: What recommendations do you have for future research as a result of this study?

• Dr. Sullivan: The identified genes are involved in brain development and our next task is to understand these genes.

• Citation:

• Neuroscience 14 abstract discussing:

• Mother’s soothing presence makes pain go away, changes gene activity in infant brain

Read the rest of the interview on MedicalResearch.comContent Not Intended as Specific Medical Advice

Page 10: MedicalResearch.com:  Medical Research Exclusive Interviews November 26 2014

Fat Around the Heart Increases Risk of Atrial FibrillationMedicalresearch.com Interview with:

Mark Rabbat, MDAssistant Professor of Medicine and Radiology

Division of Cardiology at Loyola University Medical Center

• MedicalResearch.com: What are the main findings of this study?

• Dr. Rabbat : Atrial fibrillation (AF) is the most common sustained arrhythmia in clinical practice and is responsible for significant morbidity and mortality. Epidemiologic data suggest that obesity as measured by one’s BMI is a risk factor for Atrial fibrillation. But there is more to the story. What may be even more important than overall BMI is how much fat you have around the heart. There are many individuals who, as measured by their BMI, are not considered obese, but they have high volumes of fat around their heart, which may have been a major cause for their atrial fibrillation. Therefore, simple measures such as BMI may fail to completely inform us of a patient’s true CV risk.

• In a previous study we found that epicardial adipose tissue (EAT), or the fat around the heart, has been associated with the presence and severity of AF independent of known risk factors and body mass index (BMI). The inflammatory mediators released from the fat are metabolically active, and may promote fibrosis in adjacent heart muscle. Fibrosis of the left atrium appears to be a hallmark feature of atrial fibrillation and higher amounts of fibrosis are linked to recurrence of atrial fibrillation. New innovations in cardiac magnetic resonance imaging (CMR) allow LA fibrosis and EAT to be precisely quantified. Our current study is the first of its kind to demonstrate the association of EAT volume and extent of LA fibrosis in human AF independent of LA size, BMI, and other AF risk factors.

Read the rest of the interview on MedicalResearch.comContent Not Intended as Specific Medical Advice

Page 11: MedicalResearch.com:  Medical Research Exclusive Interviews November 26 2014

Fat Around the Heart Increases Risk of Atrial FibrillationMedicalresearch.com Interview with:

Mark Rabbat, MDAssistant Professor of Medicine and Radiology

Division of Cardiology at Loyola University Medical Center

• MedicalResearch.com: What should patients and health care providers take away from this report?

• Dr. Rabbat For many years, EAT was felt to be inert. However, this is not the case as epicardial adipose tissue is a metabolically active organ and by means of various adipokines and inflammatory mediators likely contributes to both the initiation (trigger) and maintenance (substrate) of AF.

• Assessment of EAT in future large follow-up studies are needed to further clarify the pathophysiology behind these intriguing findings and to further investigate its role in AF and other cardiovascular disease states (CAD and CHF).

• Weight loss either through healthy lifestyle, diet and exercise or via bariatric surgery has demonstrated reduction in EAT. Our current CV therapeutic armamentarium fails to target the adipose tissue. EAT may be a novel target to reduce the risk of AF and its recurrence.Citation:

• Citation:

• Abstract Presented at American Heart Association 2014

• Epicardial adipose tissue volume predicts extent of left atrial fibrosis in patients with atrial fibrillation.”

• Other co-authors of the study, all at Loyola, are Bassel Sayegh, MD; Andrew Stiff, MD; Brian Vetter, MS; Kim Chan, MBBS; Tonye Teme, MD; and senior author David Wilber, MD.

Read the rest of the interview on MedicalResearch.comContent Not Intended as Specific Medical Advice

Page 12: MedicalResearch.com:  Medical Research Exclusive Interviews November 26 2014

Benefits of Medication Therapy Management Remain UnclearMedicalResearch.com Interview with:

Meera Viswanathan, PhDRTI International, Research Triangle Park

North Carolina

• Medical Research: What is the background for this study? What are the main findings?• Dr. Viswanathan: Medications, when used appropriately, can alleviate symptoms. Often, however,

they result in side effects, interact with one another, are prescribed incorrectly, or are taken incorrectly. These problems are particularly pronounced for the elderly who may have multiple chronic conditions and may be on numerous medications. We evaluated a variety of research studies and program evaluations regarding a distinct type of health care service known as Medication Therapy Management or “MTM.” The goals of MTM services are to help patients and their clinicians to optimize prescription and nonprescription drug regimens, thereby achieving better health outcomes from drug therapy, and. at the same time to minimize the potential for harms, such as incorrect dosing and duplicate medications. Some have proposed that optimizing drug regimens and preventing adverse drug events may reduce health-care-related costs.

• Medication Therapy Management services are most often provided directly to patients by pharmacists. Sometimes the same pharmacists who dispense medications to patients offer Medication Therapy Management services as well; in other models, pharmacists working in a nondispensing role within a health care system, health insurance plan administering a prescription drug benefit program, or a centralized pharmacy call center may offer such services Although Medication Therapy Management can vary quite substantially in specifics, Medication Therapy Management programs in general share common elements; these include medication therapy review of all prescription drugs, over-the-counter products, and herbal or dietary supplements; patient education and counseling to solve issues with the drug regimen that a patient may be experiencing, such as side effects or difficulty remembering to take medications; and coordination and communication with the prescribing provider. The Medicare Prescription Drug, Improvement, and Modernization Act of 2003 allowed Medicare to expand access to Medication Therapy Management services for selected patients through Medicare Part D prescription drug benefits.

Read the rest of the interview on MedicalResearch.comContent Not Intended as Specific Medical Advice

Page 13: MedicalResearch.com:  Medical Research Exclusive Interviews November 26 2014

Benefits of Medication Therapy Management Remain UnclearMedicalResearch.com Interview with:

Meera Viswanathan, PhDRTI International, Research Triangle Park

North Carolina

Through a rigorous and systematic review of health care research literature, we identified 44 studies that compared one or more groups of patients who received Medication Therapy Management services with a group of patients who did not. In some studies, patients were randomly assigned to the groups; for others, patients “opted in” to receive services through a prescription drug benefit or other program available to them as part of their usual health care. The investigators reported their findings using a diverse set of outcomes: impacts on health (e.g., symptoms, quality of life, adverse drug events, death), on quality of care (e.g., blood pressure and diabetes control, medication-related problems), or on costs and health care use (e.g., hospital admissions, clinic visits, drug costs). For most of the outcomes we evaluated, evidence was insufficient to draw conclusions about effectiveness, because the studies yielded inconsistent results or had too small sample sizes. We found adequate evidence to determine that Medication Therapy Management improves a few outcomes, such as medication appropriateness and adherence, but it has no impact on others, such as health-related quality of life and patient satisfaction. The strength of these conclusions is considered “low.”

Read the rest of the interview on MedicalResearch.comContent Not Intended as Specific Medical Advice

Page 14: MedicalResearch.com:  Medical Research Exclusive Interviews November 26 2014

Benefits of Medication Therapy Management Remain UnclearMedicalResearch.com Interview with:

Meera Viswanathan, PhDRTI International, Research Triangle Park

North Carolina

• Medical Research: What should clinicians and patients take away from your report?

• Dr. Viswanathan: As researchers, we find it challenging to provide an actionable message to clinicians and patients on reports such as this one, where the findings are largely inconsistent and Medication Therapy Management is not yet a proven strategy for accomplishing the many goals set for it. Safe and effective use of medications for all populations is critically important, and we need to find efficient, practical ways to accomplish this. Medication Therapy Management is one possible strategy, but more needs to be understood about the successful elements of the various models now in play. We hope that our report increases awareness about Medication Therapy Management services, so that all parties can make informed decisions about whether such programs are likely to bring meaningful benefits to both patients and clinicians.

Read the rest of the interview on MedicalResearch.comContent Not Intended as Specific Medical Advice

Page 15: MedicalResearch.com:  Medical Research Exclusive Interviews November 26 2014

Benefits of Medication Therapy Management Remain UnclearMedicalResearch.com Interview with:

Meera Viswanathan, PhDRTI International, Research Triangle Park

North Carolina

• Dr. Viswanathan: Because Medication Therapy Management studies to date have varied widely in their design and implementation, they understandably produce inconsistent results. As a result, the benefits of Medication Therapy Management are yet to be proven. Investigators reporting on newer studies should describe MTM service components and report on fidelity of service delivery. This information will help to mitigate the problem of comparing markedly dissimilar programs or trying to combine information from such studies. Attention to design and full reporting can also contribute to knowledge about how the effectiveness of Because Medication Therapy Management studies to date have varied widely in their design and implementation, they understandably produce inconsistent results. may vary by component or the context of implementation of such programs. Because of the complexity of these kinds of interventions and the fact that Because Medication Therapy Management studies to date have varied widely in their design and implementation, they understandably produce inconsistent results. programs are already in widespread practice, we believe that future investigations and evaluations need to move beyond the “does it work?” issues to research questions that explore the “how,” “why,” and “where” of MTM programs.

• Citation:

• Viswanathan M, Kahwati LC, Golin CE, et al. Medication Therapy Management Interventions in Outpatient Settings: A Systematic Review and Meta-analysis. JAMA Intern Med. Published online November 17, 2014. doi:10.1001/jamainternmed.2014.5841.

Read the rest of the interview on MedicalResearch.comContent Not Intended as Specific Medical Advice

Page 16: MedicalResearch.com:  Medical Research Exclusive Interviews November 26 2014

Flu Season Linked To Increase in Acute Aortic DissectionMedicalResearch.com Interview with:

Harleen Sandhu, MD MPHSenior Researcher

University of Texas Health Science Center

• Medical Research: What is the background for this study? What are the main findings?

Dr. Sandhu: Previous studies have shown a correlation between seasonal variations and occurrence of acute aortic dissection, however, reasons for such associations are unknown. Seasonal flu activity has been associated with the occurrence of cardiovascular diseases such as acute myocardial infarction in the past. This led us to verify this seasonal correlation in our experience with acute aortic dissection patients and to further investigate if its incidence was associated with flu activity. Our results confirmed the seasonal variation in acute aortic dissection as well as demonstrated a positive correlation with seasonal flu activity.

• Medical Research: What should clinicians and patients take away from your report?

Dr. Sandhu: Our study was designed to demonstrate a correlation between acute aortic dissection incidence and flu activity, if any, however currently we do not fully understand this association and further research is needed. Given this limitation, we cannot make strong recommendations at the present time. However, people with a family history of aortic dissection or connective tissue disorders such as Marfan syndrome should discuss flu shots with their physicians. Additionally, clinicians should adopt a high index of suspicion for acute aortic dissection in patients who present with usual signs and symptoms indicative of aortic dissection especially during the months of November – March and when the flu activity is high.

Read the rest of the interview on MedicalResearch.comContent Not Intended as Specific Medical Advice

Page 17: MedicalResearch.com:  Medical Research Exclusive Interviews November 26 2014

Flu Season Linked To Increase in Acute Aortic DissectionMedicalResearch.com Interview with:

Harleen Sandhu, MD MPHSenior Researcher

University of Texas Health Science Center

• Medical Research: What recommendations do you have for future research as a result of this study?

Dr. Sandhu: The current study demonstrated a correlation between the incidences of acute aortic dissection and flu but was not designed to analyze the pathophysiology of this association. However, our results indicate that further mechanistic studies are required to elucidate the causative pathways underlying this association. At this stage we can only speculate that the flu creates an inflammatory reaction that could theoretically increase chances of dissection in susceptible individuals; however, this theory needs to be verified. Furthermore, the utilization and effects of flu vaccination needs to be examined as well.

• AHA 14 abstract discussing:

• High hospital admissions for acute aortic dissection coincide with peak flu season

Read the rest of the interview on MedicalResearch.comContent Not Intended as Specific Medical Advice

Page 18: MedicalResearch.com:  Medical Research Exclusive Interviews November 26 2014

Musical Training Enhances Long Term MemoryMedicalResearch.com Interview Invitation with:

Dr. Heekyeong ParkAssistant Professor of PsychologyUniversity of Texas at Arlington

• Medical Research: What is the background for this study? What are the main findings?

• Dr. Park: This study shows that music experts with extensive musical training may have altered neural processing related to improved memory.

• There has been much interest in the beneficial effects of musical training on cognition. Notably, musical training has been reported to boost processing of verbal material. Previous studies have indicated that musical training was related to superior verbal working memory and that these differences in musical training were associated with differences in neural activity in brain regions important for verbal processing. However, it was not clear whether musical training impacts memory in general, beyond working memory for verbal items. By recruiting professional musicians with vast instrumental training, we investigated if extensive musical training has a broad impact on memory with corresponding changes in the brain.

• For this study, we compared highly trained musicians (10+ years of experience) and individuals with little or no formal musical training on working memory and long-term memory tasks. Each memory task included both verbal and pictorial items. We measured memory accuracy on tasks and scalp-recorded changes in the brain’s electrical activity (ERPs) while participants studied and remembered items. Musicians showed enhanced performance on the working memory task for both words and pictures. For the long-term memory task, musicians also remembered studied pictures better than non-musicians. These behavioral findings demonstrate the relationship between extensive musical training and improved memory broadly. ERP waveforms were also different between musicians and non-musicians while they performed long-term memory tasks.

Read the rest of the interview on MedicalResearch.comContent Not Intended as Specific Medical Advice

Page 19: MedicalResearch.com:  Medical Research Exclusive Interviews November 26 2014

Musical Training Enhances Long Term MemoryMedicalResearch.com Interview Invitation with:

Dr. Heekyeong ParkAssistant Professor of PsychologyUniversity of Texas at Arlington

• Medical Research: What should clinicians and patients take away from your report?

• Dr. Park: Our results indicate that cognitive benefits of musical training do not seem to be limited to working memory for verbal material. These findings suggest that extensive musical training is associated with improved long-term memory beyond working memory. Further, different ERP activity between musicians and non-musicians implicates the possibility that extensive musical training may alter memory processing in the brain. In conclusion, these findings highlight the beneficial effects of musical training on memory in general.

• Medical Research: What recommendations do you have for future research as a result of this study?

• Dr. Park: The next step for this research will include the investigation of the pattern of neural networks underlying long-term memory with musicians and individuals with varying levels of musical training. In addition, we aim to perform covariate analyses with memorial advantage due to music training and individual differences.

• Citation:

• Neuroscience 2014 abstract discussing:

• Musicians show advantages in long-term memory

• The findings presented at the Society for Neuroscience Annual Conference in Washington DC on November 18 2014. The abstract for the presentation “An ERP study of memory differences between musicians versus non-musicians” by J. SCHAEFFER, R. MEAHL, and H. PARK is available

Read the rest of the interview on MedicalResearch.comContent Not Intended as Specific Medical Advice

Page 20: MedicalResearch.com:  Medical Research Exclusive Interviews November 26 2014

Sleep Deprivation: Specific Molecular Changes Lead To Memory ImpairmentMedicalResearch.com Interview with:

Jennifer Choi Tudor, Ph.D. Postdoctoral FellowTed Abel Lab Department of Biology 10-17

Smilow Center for Translational Research Philadelphia, PA 19104

• Medical Research: What is the background for this study? What are the main findings?• Dr. Tudor: We (Dr. Tudor, Dr. Abel, and colleagues) are interested in better understanding the molecular

changes that occur with sleep deprivation. Previously, we found that the expression of over 500 genes changes with sleep deprivation and that many of the genes were involved with protein synthesis. Upon further investigation, we found that 5 hours of sleep deprivation impairs protein synthesis in the hippocampus, a brain region critical for memory. This impairment is due to changes in mammalian target of rapamycin (mTOR) signaling and eukaryotic initiation factor 4E binding protein 2 (4EBP2) is critical to this process. When we boosted levels of 4EBP2 in the hippocampus, mice that were sleep deprived were resistant to the detrimental effects of sleep deprivation on memory.

Medical Research: What should clinicians and patients take away from your report?

Dr. Tudor: Our findings show that there are specific molecular changes that accompany sleep deprivationand that these changes impair memory. Our work further underscores the importance of sleep for proper molecular signaling and subsequent brain function.

• Medical Research: What recommendations do you have for future research as a result of this study?

• Dr. Tudor: One of the big questions that remains to be answered is what are the specific proteins whose synthesis is being impacted by sleep deprivation. Once we figure out which proteins have impaired synthesis, we can potentially start thinking about development of therapeutics to address memory impairments associated with sleep loss.Citation:

• Neuroscience 14 abstract:• Boosting Production of Specific Brain Protein Reduces Memory Loss from Sleep Production

Read the rest of the interview on MedicalResearch.comContent Not Intended as Specific Medical Advice

Page 21: MedicalResearch.com:  Medical Research Exclusive Interviews November 26 2014

Rapidly Digested Carbohydrates May Promote Development of DiabetesMedicalResearch.com Interview with:

Wenjie Ma MS Doctoral StudentHarvard School of Public Health

• Medical Research: What is the background for this study? What are the main findings?

• Response: De novo lipogenesis (DNL) is the process whereby excess carbohydrate and protein are converted into saturated fatty acids (SFAs) and monounsaturated fatty acids (MUFAs). Emerging animal and in vitro evidence suggests that DNL might play an important role in metabolic regulation and influence the pathogenesis of type 2 diabetes. We used circulating biomarkers SFAs and MUFAs to investigate the prospective associations with incident diabetes in the Cardiovascular Health Study, a community-based cohort of older US adults. We found that circulating palmitic acid and stearic acid were associated with higher risk of incident diabetes, whereas vaccenic acid was associated with lower risk. In contrast, dietary intakes of saturated fatty acids and monounsaturated fatty acids were not associated with diabetes risk.

• Medical Research: What should clinicians and patients take away from your report?

• Response: Our findings, together with the likely harms of rapidly digested dietary carbohydrate on the development of diabetes, provide support for reducing the consumption of rapidly digested carbohydrate to prevent diabetes. The discordant results for circulating concentrations and dietary intakes facilitate the inference on the potentially different effects of metabolic compared with dietary exposure to the fatty acids in the DNL pathway.

Read the rest of the interview on MedicalResearch.comContent Not Intended as Specific Medical Advice

Page 22: MedicalResearch.com:  Medical Research Exclusive Interviews November 26 2014

Rapidly Digested Carbohydrates May Promote Development of DiabetesMedicalResearch.com Interview with:

Wenjie Ma MS Doctoral StudentHarvard School of Public Health

• Medical Research: What recommendations do you have for future research as a result of this study?

• Response: Our study highlights the need to further investigate the biological mechanisms that may link these specific fatty acids in the DNL pathway to the pathogenesis of diabetes.

• Citation:

• Prospective association of fatty acids in the de novo lipogenesis pathway with risk of type 2 diabetes: the Cardiovascular Health StudyWenjie Ma,et al

• Am J Clin Nutr 2015 ajcn.092601; First published online November 12, 2014. doi:10.3945/ajcn.114.092601

Read the rest of the interview on MedicalResearch.comContent Not Intended as Specific Medical Advice

Page 23: MedicalResearch.com:  Medical Research Exclusive Interviews November 26 2014

Heart Attack Rates Drop Nearly 25%MedicalResearch.com Interview with:

Michael J. Ward, MD, MBAAssistant Professor Vanderbilt University

Department of Emergency Medicine Nashville, TN 37232

• Medical Research: What is the background for this study? What are the main findings?

• Dr. Ward: The number of Americans living with cardiovascular disease is only expected to increase in the coming years. However, we do not know the national effects of increased medication use and preventive efforts to stop the most serious form of a heart attack, called an ST-elevation myocardial infarction (STEMI). In particular, there are no estimates of how often this serious form of a heart attack shows up in the emergency department.

• Between 2006 and 2011 we found an average of 258,000 STEMIs annually in the U.S. or 8.7 per 10,000 U.S. adults per year. Interestingly, the number of STEMIs has decreased by more than 70,000 per year over this time, a 24% reduction. We found similar annual decreases across every age group and geographic region in the U.S. The decreases were most pronounced among those 85 years and older and in the Midwest.

• Medical Research: What should clinicians and patients take away from your report?

• Dr. Ward: These are the first national estimates of STEMI presentation to U.S. emergency departments and we found that emergency department visits for STEMI are decreasing across the board. This is very good news for patients. Prevention (in the form of medication) likely plays a central role in our findings. As fewer STEMIs present to U.S. emergency departments, this could change the role emergency departments play in the management of this time-sensitive condition. Clinical providers both in the emergency department and cardiology may also be less experienced in caring for patients with these severe heart attacks as fewer patients present to U.S. hospitals with a STEMI.

Read the rest of the interview on MedicalResearch.comContent Not Intended as Specific Medical Advice

Page 24: MedicalResearch.com:  Medical Research Exclusive Interviews November 26 2014

Heart Attack Rates Drop Nearly 25%MedicalResearch.com Interview with:

Michael J. Ward, MD, MBAAssistant Professor Vanderbilt University

Department of Emergency Medicine Nashville, TN 37232

• Medical Research: What recommendations do you have for future research as a result of this study?

• Dr. Ward: Future studies should focus on how the emergency department’s role changes as the number of STEMIs decrease. Further exploration is also needed to understand how scarce healthcare dollars should be allocated-to acute emergency care or to preventive efforts. Last, does the decrease in number of STEMIs affect clinical training and experience for physicians who must care for patients with STEMI and does that affect patient outcomes.

• Citation:

• American Journal of Cardiology and presented at the national American Heart Association meeting in Chicago this November 2014.

Read the rest of the interview on MedicalResearch.comContent Not Intended as Specific Medical Advice

Page 25: MedicalResearch.com:  Medical Research Exclusive Interviews November 26 2014

Study Enhances Understanding of Genetic Mechanisms of MemoryMedicalResearch.com Interview with: Vijay Ramanan, PhD

Indiana University Center for Neuroimaging (CfN)Department of Radiology

Indianapolis, IN 46202

• Medical Research: What is the background for this study? What are the main findings?• Dr. Ramanan: Impairment in episodic memory is one of the first clinical deficits in early Alzheimer’s

disease, the most common cause of dementia. Among other examples, this might be reflected as an inability to recall an article recently read or as difficulty remembering what one had for dinner last night. Unfortunately, the genetic and environmental mechanisms underlying these deficits are not fully understood. Our goal was to discover new genes and pathways underlying memory performance to help identify potential drug targets for protecting against and ultimately reversing memory loss in dementia and normal aging.

• Through studying a large representative sample of older Americans, we discovered a variant (single nucleotide polymorphism or SNP) in the FASTKD2 gene associated with better memory performance and replicated this finding in independent samples. We then integrated additional data to extend our understanding of the effect of this SNP. For example, we know that the hippocampus is a vital brain structure for encoding and retrieving memories and it is well-understood that decreased hippocampal volume is a key early marker of Alzheimer’s disease and one that can be measured noninvasively through magnetic resonance imaging (MRI). We predicted that this new memory-protective SNP would be associated with increased hippocampal volume and this turned out to be true. We also discovered that carriers of this memory-protective SNP exhibited lower levels of proteins involved in cell death in the cerebrospinal fluid bathing the brain and spinal cord, a striking finding given that FASTKD2 encodes a protein that appears to promote apoptosis (i.e., programmed cell death). Together, these convergent findings are consistent with a neuroprotective effect of this novel SNP discovery. More broadly, our results nominate FASTKD2and its functional pathways as potential targets for modulating neurodegeneration to combat memory loss in older adults.

Read the rest of the interview on MedicalResearch.comContent Not Intended as Specific Medical Advice

Page 26: MedicalResearch.com:  Medical Research Exclusive Interviews November 26 2014

Study Enhances Understanding of Genetic Mechanisms of MemoryMedicalResearch.com Interview with: Vijay Ramanan, PhD

Indiana University Center for Neuroimaging (CfN)Department of Radiology

Indianapolis, IN 46202

Medical Research: What should clinicians and patients take away from your report?

• Dr. Ramanan: To our knowledge this represents the largest genetic study of human memory to date and includes more than 14,000 older adults from 6 independent cohorts centered at numerous institutions across North America and Europe. The greatest value of this study is in how it enhances our understanding of the genetic mechanisms driving memory functioning. Particularly given that our top discovery was protective, we will be interested to follow whether drug targeting of this gene and pathway may be used to protect against or reverse neuronal dysfunction and memory loss. Given the rising incidence and burdens of dementia, discovering more of the factors underlying what is frequently described as the “Alzheimer’s epidemic” will be crucial for predicting risk and developing effective treatments.

Read the rest of the interview on MedicalResearch.comContent Not Intended as Specific Medical Advice

Page 27: MedicalResearch.com:  Medical Research Exclusive Interviews November 26 2014

Study Enhances Understanding of Genetic Mechanisms of MemoryMedicalResearch.com Interview with: Vijay Ramanan, PhD

Indiana University Center for Neuroimaging (CfN)Department of Radiology

Indianapolis, IN 46202

• Medical Research: What recommendations do you have for future research as a result of this study?

• Dr. Ramanan: Functional genomics experiments using animal and cultured cell model systems will be valuable for directly testing the impact of drugs targeting FASTKD2 and its functional pathways. We would also like to study additional large datasets not analyzed in this report, particularly to assess other genes showing some signal in our analysis, such as MTOR which is a major active drug target for Alzheimer’s due its known roles in amyloid and tau pathology. Research using next generation sequencing data, which provides much more detailed views of DNA and RNA, will also help to characterize the mechanism of action for the SNPs nominated in this study and may identify other nearby variants which were too rare to identify in our analyses.

• Citation:

• FASTKD2 is associated with memory and hippocampal structure in older adults

• V K Ramanan, K Nho, L Shen, S L Risacher, S Kim, B C McDonald, M R Farlow, T M Foroud, S Gao, H Soininen, I Kłoszewska, P Mecocci, M Tsolaki, B Vellas, S Lovestone, P S Aisen, R C Petersen, C R Jack, L M Shaw, J Q Trojanowski, M W Weiner, R C Green, A W Toga, P L De Jager, L Yu, D A Bennett, A J Saykin and for the Alzheimers Disease Neuroimaging Initiative (ADNI)27Molecular Psychiatry , (11 November 2014) | doi:10.1038/mp.2014.142

Read the rest of the interview on MedicalResearch.comContent Not Intended as Specific Medical Advice

Page 28: MedicalResearch.com:  Medical Research Exclusive Interviews November 26 2014

Flu Vaccination May Be Effective Even When Circulating Strain Different From VaccineMedicalResearch.com Interview with:

Maryam Darvishian MScDepartment of Epidemiology, University Medical Center Groningen,

Prof Edwin R van den Heuvel Department of Epidemiology, University Medical Center Groningen,University of Groningen, Groningen, NetherlandsDepartment of Mathematics and

• Medical Research: What is the background for this study?

• Reply: In most developed countries, seasonal influenza vaccine is the standard care for elderly people, but there exists still discussions on whether vaccination is effective. Conducting RCT is not considered ethical and thus the main body of evidence comes from observational studies. Unfortunately, these studies (e.g. cohort studies) are susceptible to different sources of biases especially selection bias which makes it difficult to judge the effectiveness. In recent years test-negative design (TND) studies has been designed. It is a special type of case-control study which would limit the bias, due to similar health care-seeking behavior in cases and controls. The current study is a meta-analysis of TND case-control studies. It is the first meta-analysis of this type of studies and also the first meta-analysis that combined 35 studies for estimation of influenza vaccine effectiveness. More specifically, the meta-analysis assesses the influenza vaccine effectiveness against laboratory-confirmed influenza (LCI) among the elderly population.

• Medical Research: What are the main findings?

• Reply: Our findings show that influenza vaccine is effective against laboratory-confirmed influenza in community-dwelling elderly during regional and widespread influenza outbreaks. In cases where the vaccine matches the circulating virus, the vaccination is most effective, but it is also effective when the circulating virus does not match the strains in the vaccine.

Read the rest of the interview on MedicalResearch.comContent Not Intended as Specific Medical Advice

Page 29: MedicalResearch.com:  Medical Research Exclusive Interviews November 26 2014

Flu Vaccination May Be Effective Even When Circulating Strain Different From VaccineMedicalResearch.com Interview with:

Maryam Darvishian MScDepartment of Epidemiology, University Medical Center Groningen,

Prof Edwin R van den Heuvel Department of Epidemiology, University Medical Center Groningen,University of Groningen, Groningen, NetherlandsDepartment of Mathematics and

• Medical Research: What should clinicians and patients take away from your report?

• Reply: Based on our findings, we believe that influenza vaccination uptake should increase among the elderly population.

• Medical Research: What recommendations do you have for future research as a result of this study?

• Reply: Influenza vaccine effectiveness is estimated annually. However, most of the studies do not adjust for covariates such as health status and possible co-morbidities in elderly population. Moreover, there is no individual patient data (IPD) meta-analysis that would take into account these important covariates and possibly provide a more accurate estimate of vaccine effectiveness. We believe that such studies should be conducted in the near future to verify the current estimates. Additionally, vaccines would improve when they would match the circulating virus, but viruses mutate and predicting the new strain is difficult. Vaccines that would improve on the match rate would be beneficial as well.

• Citation:

• Effectiveness of seasonal influenza vaccine in community-dwelling elderly people: a meta-analysis of test-negative design case-control studiesMaryam Darvishian MSc,Maarten J Bijlsma MSc,Eelko Hak PhD,Prof Edwin R van den Heuvel PhDThe Lancet Infectious Diseases – 1 December 2014 ( Vol. 14, Issue 12, Pages 1228-1239 ) DOI: 10.1016/S1473-3099(14)70960-0

Read the rest of the interview on MedicalResearch.comContent Not Intended as Specific Medical Advice

Page 30: MedicalResearch.com:  Medical Research Exclusive Interviews November 26 2014

Addictions May Predispose To Excessive Gestational Weight GainMedicalResearch.com Interview with:

Michele D. Levine Ph.D.Associate Professor of Psychiatry and Psychology Western Psychiatric Institute and Clinic

Department of Statistics, University of Pittsburgh, Pittsburgh PA

• Medical Research: What is the background for this study? What are the main findings?

• Dr. Levine: Many women quit smoking as a result of pregnancy. However, psychiatric disorders, which are prevalent among smokers can contribute to weight gain. Thus, we sought to examine the relationship between maternal psychiatric disorders and gestational weight gain in a sample of pregnant former smokers. Results from the present study demonstrate that the rates of psychiatric disorders were high among pregnant former smokers and that more than half of women gained more weight than recommended by the IOM. Although a history of having had any psychiatric disorder was not associated with gestational weight gain, a history of alcohol use disorder specifically was positively related to gestational weight gain.

• Medical Research: What should clinicians and patients take away from your report?

• Dr. Levine: The high rates of lifetime psychiatric disorders and excessive gestational weight gain suggest that young, pregnant former smokers may be a vulnerable population in need of specialized or targeted interventions to sustain positive health related behavior changes. In addition, the link between increased gestational weight gain and a history of alcohol use disorder suggests that women with a vulnerability to addiction may be at particular risk of gaining more weight in pregnancy. Although this finding warrants further study, such women may benefit from treatment efforts that focus on improving self-regulation.

Read the rest of the interview on MedicalResearch.comContent Not Intended as Specific Medical Advice

Page 31: MedicalResearch.com:  Medical Research Exclusive Interviews November 26 2014

Addictions May Predispose To Excessive Gestational Weight GainMedicalResearch.com Interview with:

Michele D. Levine Ph.D.Associate Professor of Psychiatry and Psychology Western Psychiatric Institute and Clinic

Department of Statistics, University of Pittsburgh, Pittsburgh PA

• Medical Research: What recommendations do you have for future research as a result of this study?

• Dr. Levine: Characterizing women who may be at risk of excessive gestational weight gain has important public health implications, and findings from the present study provide initial evidence documenting high rates of psychiatric disorder and excess gestational weight gain among pregnant former smokers. Thus, additional research is needed to better understand psychosocial factors that are predictive of positive health behaviors during pregnancy.

• Citation:Psychiatric Disorders and Gestational Weight Gain among Women Who Quit Smoking During PregnancyLevine, Michele D. et al.

• Journal of Psychosomatic ResearchReceived: May 16, 2014; Received in revised form: November 10, 2014; Accepted: November 12, 2014; Published Online: November 15, 2014

• DOI: http://dx.doi.org/10.1016/j.jpsychores.2014.11.013

Read the rest of the interview on MedicalResearch.comContent Not Intended as Specific Medical Advice

Page 32: MedicalResearch.com:  Medical Research Exclusive Interviews November 26 2014

US Skin Cancer Costs Top $8 Billion AnnuallyMedical Research.com Interview with:

Gery P. Guy Jr., PhD, MPH, Health economistCDC: Division of Cancer Prevention and Control’s Epidemiology and Applied Research Branch.

• Medical Research: What is the background for this study?

• Dr. Guy: Skin cancer is the most commonly diagnosed cancer in the United States and is a growing public health problem. Melanoma, the deadliest form of skin cancer, is responsible for more than 12,000 deaths each year and is diagnosed in over 70,000 people per year. The number of skin cancer cases continues to increase every year, however little is known about the economic burden of treatment. The purpose of our study was to examine trends in the number of people treated for skin cancer and the cost of treatment.

• Medical Research: What are the main findings?

• Dr. Guy: Each year in the United States, nearly 5 million people are treated for skin cancer, at a cost of $8.1 billion dollars. The costs associated with skin cancer treatment are not only rising, but they are increasing substantially more than for other cancers. The average annual cost for skin cancer treatment increased from $3.6 billion during 2002-2006, to $8.1 billion during 2007-2011, an increase in costs of 126 percent while the average annual cost for treatment of all other cancers increased by 25 percent. The average annual number of adults treated for skin cancer increased from 3.4 million during 2002-2006 to 4.9 million in 2007-2011.

Read the rest of the interview on MedicalResearch.comContent Not Intended as Specific Medical Advice

Page 33: MedicalResearch.com:  Medical Research Exclusive Interviews November 26 2014

US Skin Cancer Costs Top $8 Billion AnnuallyMedical Research.com Interview with:

Gery P. Guy Jr., PhD, MPH, Health economistCDC: Division of Cancer Prevention and Control’s Epidemiology and Applied Research Branch.

• Medical Research: What should clinicians and patients take away from your report? • Dr. Guy: Most cases of cases of skin cancer, including melanoma, are caused by exposure to ultraviolet (UV) light. Therefore

many skin cancers could be prevented if people avoid UV exposure that could lead to skin cancer by protecting their skin from the sun and avoiding indoor ` tanning. Clinicians can play an important role in skin cancer prevention. The US Preventive Services Task Force recommends behavioral counseling in clinical settings about minimizing UV exposure to reduce skin cancer risk for fair-skinned individuals aged 10-24 years.

• Patients are encouraged to follow the recommended steps for protecting their skin from UV rays:• Stay in the shade, especially during midday hours.• Wear clothing that covers your arms and legs.• Wear sunglasses that block both UVA and UVB rays.• Use sunscreen with SPF 15 or higher and both UVA and UVB protection.• Avoid indoor tanning.• Medical Research: What recommendations do you have for future research as a result of this study? • Dr. Guy: While this study demonstrates the substantial costs of skin cancer treatment, it also highlights the potential for

savings through prevention efforts. Skin cancer prevention efforts have been shown to reduce skin cancer incidence, mortality, and healthcare expenditures. Continued surveillance is needed to monitor the impact of skin cancer prevention efforts on the economic burden of skin cancer in the United States.

• Citation:• Prevalence and Costs of Skin Cancer Treatment in the U.S., 2002−2006 and 2007−2011

Guy, Gery P. et al.American Journal of Preventive MedicinePublished Online: November 09, 2014DOI: http://dx.doi.org/10.1016/j.amepre.2014.08.036

Read the rest of the interview on MedicalResearch.comContent Not Intended as Specific Medical Advice

Page 34: MedicalResearch.com:  Medical Research Exclusive Interviews November 26 2014

Hospital Readmissions May Not Be A Good Quality IndicatorMedicalResearch.com Interview with: Ralitza P. Parina, MPH, Senior medical student

Department of Surgery, University of California, San Diego, CAJohn Rose, MD MPH

Department of Surgery, State University of New York, Stony Brook, NY

• Medical Research: What is the background for this study? What are the main findings?

• Response: This study looked at the association between hospital 30-day readmission rates and 30-day mortality rates. While readmission rates are coming into increasing focus with CMS reimbursement cuts for hospitals with higher than expected rates, they remain a poorly studied metric of quality. High readmission rates have been unequivocally tied to increased costs, but it remains unclear whether they actually represent poor quality of care and worse outcomes for patients. We chose to compare readmission rates as a quality metric to the well-established “gold standard” of mortality.

• We found that 85% of hospitals did not show a correlation between readmission and mortality, i.e. their rates were not both high or both low. Furthermore, among hospitals that were outliers in at least one of the measures, almost a third were in the category of low or normal readmission rates with higher than expected mortality.

• The implications are twofold: first, readmission and mortality rates are not strongly correlated.Second, focusing on readmission rates as an outcome will miss a large number of poorly performing hospitals with higher than expected mortality rates but low or expected readmissions.

• Medical Research: What should clinicians and patients take away from your report?

• Response: This work really has broader policy implications. While a focus on reducing readmission rates will result in cost savings, it may not be the best indicator of hospital quality. As such, it may not be the most appropriate metric to tie to reimbursement. Further research needs to be done to better characterize and attempt to resolve this discrepancy.

Read the rest of the interview on MedicalResearch.comContent Not Intended as Specific Medical Advice

Page 35: MedicalResearch.com:  Medical Research Exclusive Interviews November 26 2014

Hospital Readmissions May Not Be A Good Quality IndicatorMedicalResearch.com Interview with: Ralitza P. Parina, MPH, Senior medical student

Department of Surgery, University of California, San Diego, CAJohn Rose, MD MPH

Department of Surgery, State University of New York, Stony Brook, NY

• Medical Research: What recommendations do you have for future research as a result of this study?

• Response: Future research should focus on characterizing what aspect of quality or patient care readmission rates are measuring. It has been postulated that they are a reflection of suboptimal discharge planning or a consequence of the physiological and psychological stress that occurs after hospitalization termed ‘post-hospital syndrome’.

• Furthermore, we contend that development of new quality metrics should be validated by correlation to an established “gold standard”, whether mortality or another well-defined and equivocally important outcome.

• Citation:

• Is a Low Readmission Rate Indicative of a Good Hospital?Journal of the American College of Surgeons

• Available online 8 November 2014Presented at the Western Surgical Association 121st Scientific Session, Salt Lake City, UT, November 2013

• Ralitza P. Parina, MPH,David C. Chang, PhD, MPH, MBA, John Rose, MD MPH Mark A. Talamini, MD, FACS

Read the rest of the interview on MedicalResearch.comContent Not Intended as Specific Medical Advice

Page 36: MedicalResearch.com:  Medical Research Exclusive Interviews November 26 2014

Lower Calorie Diets May Slow Aging and Memory LossMedicalResearch.com Interview with:

Stephen D. Ginsberg, Ph.D., Associate ProfessorDepartments of Psychiatry and Physiology & Neuroscience

New York University Langone Medical Center Center for Dementia ResearchNathan Kline Institute Orangeburg, NY 10962

• Medical Research: What is the background for this study? What are the main findings?

• Dr. Ginsberg: We tested the hypothesis that long-term calorie restriction positively alters gene expression within the hippocampus, a critical learning and memory area vulnerable in aging and Alzheimer’s disease. To test this hypothesis, we conducted experiments on female mice that were given food pellets 30% lower in calories than what was fed to the control group. The mice ate fewer calories derived from carbohydrates. Analyses were performed on mice in middle and old age to assess any differences in gene expression over time. Our data analysis revealed that the mice that were fed a lower calorie diet had fewer changes in approximately 900 genes that are linked to aging and memory.

• Medical Research: What should clinicians and patients take away from your report?

• Dr. Ginsberg: Our study shows how calorie restriction practically arrests gene expression levels involved in the aging phenotype, which may be beneficial for future drug discovery as well as dietary studies to improve the healthspan. While restrictive dietary regimens have been well-known for decades to prolong the lives of rodents and other mammals, their effects in humans have not been well understood. Benefits of these diets have been touted to include reduced risk of human heart disease, hypertension, and stroke, but the widespread genetic impact on the memory and learning regions of aging brains has not before been shown with this number of genes (>10,000) being investigated simultaneously. Although the results of this study does not mean calorie restriction is the “fountain of youth,” it does add evidence for the role of diet in delaying the effects of aging and age-related disease.

Read the rest of the interview on MedicalResearch.comContent Not Intended as Specific Medical Advice

Page 37: MedicalResearch.com:  Medical Research Exclusive Interviews November 26 2014

Lower Calorie Diets May Slow Aging and Memory LossMedicalResearch.com Interview with:

Stephen D. Ginsberg, Ph.D., Associate ProfessorDepartments of Psychiatry and Physiology & Neuroscience

New York University Langone Medical Center Center for Dementia ResearchNathan Kline Institute Orangeburg, NY 10962

• Medical Research: What recommendations do you have for future research as a result of this study?

• Dr. Ginsberg: The results of these findings widens the door to study more in-depth mechanisms underlying positive benefits of calorie restriction on anti-aging genetics. It is possible that a sustained and controlled reduction in calorie intake could help slow brain aging and deter chronic disease.

• Citation:

• Neuroscience 14 abstract discussing:Calorie-Restricting Diets Slow Aging

• The study was conducted by Marissa J Schafer, PhD, Igor Dolgalev, MS, Adriana Heguy, PhD, and Stephen D Ginsberg, PhD.

Read the rest of the interview on MedicalResearch.comContent Not Intended as Specific Medical Advice

Page 38: MedicalResearch.com:  Medical Research Exclusive Interviews November 26 2014

Long Term Influence of Oral Contraceptives on MortalityMedicalResearch.com Interview with:

Karin B. Michels, ScD, PhDAssociate Professor of Obstetrics, Gynecology and Reproductive Biology

Harvard Medical School

• Medical Research: What is the background for this study? What are the main findings?

• Dr. Michels: We were interested in studying the long-term effects of oral contraceptive use on mortality. Given the widespread use of oral contraceptives, this is an important question pertaining to millions of women worldwide. We explored this question in the large Nurses’ Health Study, a cohort of 121,700 women in the US, who have been followed for 38 years. We found that oral contraceptive use does not impact overall mortality. However, breast cancer mortality was slightly increased, especially with long-term use of oral contraceptives.

Medical Research: What should clinicians and patients take away from your report?

• Dr. Michels: Oral contraceptives don’t seem to affect overall mortality. However, women with breast cancer or a family history of breast cancer warrant special consideration, especially for longer term use of oral contraceptives.

• Medical Research: What recommendations do you have for future research as a result of this study?

• Dr. Michels: We were only able to study the higher-dose first- and second generation OCs. Further studies may want to address the safety of the currently used lower-dose formulations, in particular with respect to breast cancer incidence and mortality.

• Citation:

• Charlton Brittany M, Rich-Edwards Janet W, Colditz Graham A, Missmer Stacey A, Rosner Bernard A, Hankinson Susan E et al. Oral contraceptive use and mortality after 36 years of follow-up in the Nurses’ Health Study: prospective cohort study BMJ 2014; 349:g6356

Read the rest of the interview on MedicalResearch.comContent Not Intended as Specific Medical Advice

Page 39: MedicalResearch.com:  Medical Research Exclusive Interviews November 26 2014

Patients With Type of Familial Sudden Death Need Long Term Follow UpMedicalResearch.com with:

Giulio Conte MDHeart Rhythm Management Centre

UZ-VUB Brussel, Belgium

• Medical Research: What is the background for this study? What are the main findings?• Dr. Conte: The evolution of Brugada syndrome from pediatric to adult age has not been previously evaluated. It has been

shown that the electrocardiographic phenotype of Brugada syndrome do not manifest during childhood in the large majority of cases. Drug challenge with ajmaline is recommended to unmask the diagnostic electrocardiogram in patients with family history of Brugada syndrome and normal electrocardiograms. However, the ideal age to perform such screening has not been established yet. With this study we aimed to investigate the clinical value of repeating ajmaline challenge after puberty in pediatric family members with an initial negative drug test. Repeat ajmaline challenge after puberty unmasked Brugadasyndrome in 23% of family members with a previously negative drug test. Of the newly positive patients, 30% developed symptoms, 10% ventricular fibrillation and 10% spontaneous Brugada type 1 electrocardiogram.

Medical Research: What should clinicians and patients take away from your report?• Dr. Conte: Our study highlights the existence in Brugada syndrome of an age-dependent response to sodium channel blocker

administration. These findings support the need for continuous monitoring of patients and family members with Brugadasyndrome, even those considered at low risk at a pediatric age. A family member with a normal electrocardiogram who develops symptoms after puberty should be always investigated with an ajmaline test to rule out Brugada syndrome even if he/she had a negative drug test performed before puberty.

• Medical Research: What recommendations do you have for future research as a result of this study?• Dr. Conte: Our data underline the potential role of hormones as causative factors in determining the post-pubertal response

to ajmaline in patients with the syndrome. Further specific studies conducted on larger populations of patients are needed to confirm our results.

• Citation:Conte G, de Asmundis C, Ciconte G, et al. Follow-up From Childhood to Adulthood of Individuals With Family History of Brugada Syndrome and Normal Electrocardiograms. JAMA. 2014;312(19):2039-2041. doi:10.1001/jama.2014.13752.

Read the rest of the interview on MedicalResearch.comContent Not Intended as Specific Medical Advice

Page 40: MedicalResearch.com:  Medical Research Exclusive Interviews November 26 2014

HPV Vaccination Rates Leaves Room For ImprovementMedicalResearch.com Interview with:Dr. Raquel Qualls-Hampton MD, MS

Assistant ProfessorUniversity of North Texas Health Science Center

• Medical Research: What is the background for this study? What are the main findings?

• Dr. Qualls-Hampton: There are currently two vaccines approved by the Food and Drug Administration (FDA)—Gardasil for males and Gardasil and Cervix for females – that protect against the human papilloma virus (HPV). These vaccines are recommended by the ACIP for females ages 9 to 26 years and males ages 9 to 21 years. Both vaccines protect males and females against some of the most common types of HPV. HPV vaccines are administered in three doses over six months and are considered safe and effective. However, the promise of these vaccines is going unfulfilled as initiation and completion rates for the three doses are suboptimal among females and males.

• Nationally, although HPV vaccination initiation coverage is increasing, overall vaccine completion rates are at suboptimal levels and below the U.S. Department of Health and Human Services’ Healthy People 2020 initiative target of 80%. Thus, many states are turning to legislative interventions in efforts to increase initiation and completion rates. This study examines HPV vaccination legislative initiatives and their impact, specifically in estimating state legislation’s effects on HPV vaccine initiation, completion and patient care provider recommendations by gender.

Read the rest of the interview on MedicalResearch.comContent Not Intended as Specific Medical Advice

Page 41: MedicalResearch.com:  Medical Research Exclusive Interviews November 26 2014

HPV Vaccination Rates Leaves Room For ImprovementMedicalResearch.com Interview with:Dr. Raquel Qualls-Hampton MD, MS

Assistant ProfessorUniversity of North Texas Health Science Center

• Our findings demonstrate that state legislation does influence HPV initiation and completion rates of adolescents. Moreover, this influence differs by gender. Conversely, patient care provider vaccine recommendations to parents does not significantly differ by state legislation however the relationship does vary by gender. Our study confirms and elucidates what other research has suggested: that parent HPV vaccination decisions are multifactorial, go beyond HPV legislation. Finally, there are physician factors (strong HPV vaccine recommendation) and parent-specific factors (e.g., social and cultural) that weigh heavily in HPV vaccine decisions for both males and female.

• We confident these findings would interest both researchers and the general public, especially policy makers and clinical health professionals

Read the rest of the interview on MedicalResearch.comContent Not Intended as Specific Medical Advice

Page 42: MedicalResearch.com:  Medical Research Exclusive Interviews November 26 2014

HPV Vaccination Rates Leaves Room For ImprovementMedicalResearch.com Interview with:Dr. Raquel Qualls-Hampton MD, MS

Assistant ProfessorUniversity of North Texas Health Science Center

• Medical Research: What should clinicians and patients take away from your report?

• Dr. Qualls-Hampton: Clinicians must understand the purpose of the HPV vaccine – cancer prevention – and educate their patients and parents.

• Medical Research: What recommendations do you have for future research as a result of this study?

• Dr. Qualls-Hampton: My team and I are working on the next study, which will involve assessing HPV vaccine knowledge and perception of physicians.

• Citation:

• APHA 2014 abstract discussing:

• More HPV legislation may not result in greater HPV vaccine uptake

Read the rest of the interview on MedicalResearch.comContent Not Intended as Specific Medical Advice

Page 43: MedicalResearch.com:  Medical Research Exclusive Interviews November 26 2014

More CT Scans Done For Minor Head Trauma Than Guidelines RecommendMedicalResearch.com Interview with: Dr. Jennifer Marin MD MSc

Director of Emergency UltrasoundDivision of Pediatric Emergency Medicine

Assistant Professor of Pediatrics and Emergency MedicineUniversity of Pittsburgh School of Medicine

• Medical Research: What is the background for this study? What are the main findings?

Dr. Marin: Overuse of diagnostic imaging in the emergency department has become a focus of concern from policy makers, patients, and physicians. There are evidence-based clinical decision rules and policy recommendations published in order to optimize the use of such imaging. However, physicians don’t necessarily use these tools in their decision-making. Head computed tomography (CT) imaging for patients with minor head trauma is a common CT performed in the emergency setting. Our study sought to evaluate how often physicians adhered to the American College of Emergency Physicians (ACEP) Clinical Policy on Neuroimaging. The policy outlines which patients warrant a CT in the setting of minor head trauma based on certain factors, such as age, mechanism of injury, and signs and symptoms of head trauma. What we found is that when the policy recommends that a head CT be performed, it is obtained more than 90% of the time. However, when a head CT is not recommended, it is actually obtained in nearly half of those patients. We hope this will draw attention to decision rules and clinical policies, such as that from ACEP, and remind physicians that using these tools can assist in appropriate imaging practices.

MedicalResearch.com: What further research do you recommend as a result of this work?• Dr. Marin: There are certainly many areas on which to expand our research. The next step would be to

evaluate reasons why physicians don’t adhere to these guidelines, specifically, when a CT is not recommended.

• Citation:• Adherence to a clinical decision policy for head computed tomography in adult mild traumatic brain injury

Marin, Jennifer R. et alThe American Journal of Emergency MedicinePublished Online: November 19, 2014

• DOI: http://dx.doi.org/10.1016/j.ajem.2014.11.017

Read the rest of the interview on MedicalResearch.comContent Not Intended as Specific Medical Advice

Page 44: MedicalResearch.com:  Medical Research Exclusive Interviews November 26 2014

Skin Cancer Lesions Not Affected By Agent Orange ExposureMedicalResearch.com Interview with:

Naveed Nosrati MDIndiana University School of Medicine

Staff Surgeon, Roudebush VAMC

• Medical Research: What is the background for this study? • Dr. Nosrati: We originally began this study as a broader project investigating the effect of trauma induced by biopsies on the

spontaneous clearance of a non-melanoma skin cancer. As part of that, we created a large database with many patient variables. Since we undertook this project at our local VA hospital, one of the variables available to us was Agent Orange exposure.

• Shortly after completing the study, Clemens et al published their study linking Agent Orange exposure to higher rates of invasive non-melanoma skin cancer. Their study was a pilot study of only 100 patients. As we had well over 1,000 patients, we decided to pursue a side project of how Agent Orange specifically affects our results.

• Our study was operating under the hypothesis that trauma induced by biopsies led to an inflammatory response that often led to the immunologic clearance of the remaining skin cancer. We actually coined the term “SCORCH” lesion, or spontaneous clearance of residual carcinoma histologically, for this phenomenon. With that mind, we would expect patients exposed to Agent Orange to theoretically have a more invasive form of malignancy and thus have lower rates of spontaneous clearance.

After we crunched the numbers we found that rates of clearance between those veterans exposed and not exposed to Agent Orange was the same. In fact, the rate was slightly higher in the exposed group, though this wasn’t significant. This was a very interesting result, especially in light of the other recent study. However, we can’t be too quick to jump to conclusions when comparing the studies. The two studies analyzed different variables and outcomes, and the only link is a theoretical hypothesis. What conclusion can be made though, is that SCORCH lesions aren’t affected by Agent Orange.

• Medical Research: What future research do you recommend as a result of this work?• Dr. Nosrati: Future directions of this project are to compile all the variables within the database to create a system that

better predicts when a lesion will be a SCORCH lesion. We are currently undertaking that task, as well as prospectively collecting data that puts that system to the test.

• Citation:The Effect of Agent Orange on Nonmelanoma Skin Cancer Regression RatesNosrati N, Han J, Flores R, Sood R, Tholpady SS.JAMA Surg. 2014 Nov 1;149(11):1205-6. doi: 10.1001/jamasurg.2014.953.

Read the rest of the interview on MedicalResearch.comContent Not Intended as Specific Medical Advice

Page 45: MedicalResearch.com:  Medical Research Exclusive Interviews November 26 2014

UK Drop in Antibiotic Prophylaxis Linked To Increased Number of Heart InfectionsMedicalResearch.com Interview with:

Dr. Martin Thornhill PhDDepartment of Cardiology, Taunton and Somerset NHS Trust

Taunton, Somerset, UK

• Medical Research: What is the background for this study? What are the main findings?• Dr. Thornhill: In 2008 NICE introduced controversial new guidance recommending that antibiotic

prophylaxis to prevent infective endocarditis should no longer be used. It was a rational decision, given the evidence for the effectiveness of antibiotic prophylaxis and potential concerns about costs, the development of antibiotic resistance and possible side effects from antibiotics, but it went against other guidelines from around the world that existed at the time.

• The main findings are that in England:• There has been a large and significant decline in the use of antibiotic prophylaxis.• There has been a significant increase in the number of cases of infective endocarditis, above the

baseline trend, using hospital coding data, corrected for changes in the size of the English population.

• Medical Research: What should clinicians and patients take away from your report?• Dr. Thornhill: We cannot conclude, just because there is correlation, that the change in practice has

resulted in a change in the incidence of infective endocarditis. Clinicians and patients should not change their practice at present, but should wait for further analysis of the data by guideline committees.Citation:

• Incidence of infective endocarditis in England, 2000–13: a secular trend, interrupted time-series analysisMark J Dayer PhD,Prof Simon Jones PhD,Bernard Prendergast FRCP,Prof Larry M Baddour MD,Prof Peter B Lockhart DDS,Prof Martin H Thornhill PhDThe Lancet – 18 November 2014 DOI: 10.1016/S0140-6736(14)62007-9

Read the rest of the interview on MedicalResearch.comContent Not Intended as Specific Medical Advice

Page 46: MedicalResearch.com:  Medical Research Exclusive Interviews November 26 2014

Telephone Reminders May Increase Breast Cancer ScreeningMedicalResearch.com Interview with

Dr. Jonathan MylesCentre for Cancer Prevention, Queen Mary, University of London

Wolfson Institute of Preventive Medicine, Charterhouse Square, London

• Medical Research: What is the background for this study? What are the main findings?

• Dr. Myles: Breast cancer screening uptake is low in areas of high social deprivation and large populations of some ethnic groups. The main finding of this study is that an intervention in the form of contacting women by telephone a few days before the date of their screen, reminding them of their appointment and answering any queries they may have, significantly increases uptake.

• Medical Research: What should clinicians and patients take away from your report?

• Dr. Myles: In general, there is potential for carefully planned interventions to increase uptake in areas where uptake is low. Specifically, a telephone-based intervention such as this one can improve uptake.

• Medical Research: What recommendations do you have for future research as a result of this study?

• Dr. Myles: We would like to understand more about the effect of this (and potentially other) interventions in specific ethnic and social groups.

• Citation:

• A telephone reminder intervention to improve breast screening information and access J. Offman, J. MylesPublic HealthAvailable online 18 November 2014

Read the rest of the interview on MedicalResearch.comContent Not Intended as Specific Medical Advice

Page 47: MedicalResearch.com:  Medical Research Exclusive Interviews November 26 2014

Stroke: Golden Hour Thrombolysis Improved Discharge To HomeMedicalResearch.com Interview with:

Priv.-Doz. Dr. med. Dr. phil. Martin EbingerCenter for Stroke Research Berlin (CSB)

Charité – Universitätsmedizin Berlin | CCM Berlin | Germany

• Medical Research: What is the background for this study? What are the main findings?

Response: Hitherto, little has been known about the effects of thrombolysis (tPA) in ischemic stroke within the first 60 minutes of symptom onset. That’s because the so-called golden hour thrombolysis is such a rare event. As James Grotta, Houston, Texas, recently pointed out there were only 2 patients receiving tPA within 60 minutes in the pivotal NINDS trial – both received placebo, and even the latest up-date on randomized trials of tPA includes only two further patients within 60 minutes. In our study, we used the Stroke Emergency Mobile (STEMO) for ultra-early thrombolysisin the pre-hospital setting. STEMO is a specialized ambulance equipped with a CT scanner, point-of-care laboratory, and a telemedicine connection to neuroradiologist on call. Aboard the STEMO, there is a paramedic, a radiology technician and a neurologist. The project was initiated und supervised by Heinrich Audebert from the Charité, Berlin, Germany.The main finding of our study is that we showed a statistically significant association between golden hour thrombolysis and discharge home as opposed to e.g. nursing facilities.

• Medical Research: What should clinicians and patients take away from your report?

• Response: Our report again emphasizes the importance of speedy delivery of thrombolysis in ischemic stroke. Jeff Saver from UCLA recently published US data on the impact every 15 minute delay has on clinical outcomes. Chances for a patient to leave the hospital walking drop by 4%. Or, as Atte Mertoja, Melbourne, Australia, puts it nicely in one of his recent publications: ‘Save a minute, save a day’. The STEMO turned out to be an excellent tool to increase the number of golden hour thrombolysis by the factor 10, allowing for the first time to analyze clinical effects of very early thrombolysis.

Read the rest of the interview on MedicalResearch.comContent Not Intended as Specific Medical Advice

Page 48: MedicalResearch.com:  Medical Research Exclusive Interviews November 26 2014

Stroke: Golden Hour Thrombolysis Improved Discharge To HomeMedicalResearch.com Interview with:

Priv.-Doz. Dr. med. Dr. phil. Martin EbingerCenter for Stroke Research Berlin (CSB)

Charité – Universitätsmedizin Berlin | CCM Berlin | Germany

• Medical Research: What recommendations do you have for future research as a result of this study?

• Response: So far, data from STEMO have been very convincing. They demonstrated a clear benefit in terms of time reduction from alarm to treatment. Given the most obvious association between time to treatment and functional outcome measured with the gold standard, i.e. modified Rankin Scale after 3 months, one would assume that thrombolysis in STEMO should be more effective than in-hospital thrombolysis. However, we are still lacking a study that has the power to show this over the entire 4.5 hour time window.

• Citation:

• Ebinger M, Kunz A, Wendt M, et al. Effects of Golden Hour Thrombolysis: A Prehospital Acute Neurological Treatment and Optimization of Medical Care in Stroke (PHANTOM-S) Substudy. JAMA Neurol. Published online November 17, 2014. doi:10.1001/jamaneurol.2014.3188.

Read the rest of the interview on MedicalResearch.comContent Not Intended as Specific Medical Advice

Page 49: MedicalResearch.com:  Medical Research Exclusive Interviews November 26 2014

For Weight Control, Focus on Healthy Foods, Not CaloriesMedicalResearch.com Interview with:

James J. DiNicolantonio, PharmDAssociate Editor BMJ Open HeartCardiovascular Research Scientist

Saint Luke’s Mid America Heart Institute

• Medical Research: What is the background for this study? What are the main findings?

• Dr. DiNicolantonio: Focusing on calories misdirects eating away from healthy foods (that are higher in calories – such as nuts, salmon, and avocados) and towards harmful foods (e.g. rapidly absorbable carbohydrates – including added sugars such as table sugar and high fructose corn syrup).

• Treating obesity should not focus on decreasing caloric intake, rather, it should focus on eating quality foods. Lower calorie foods – that are high in rapidly absorbable carbohydrates – drive increased hunger throughout the day, whereas higher calorie foods (such as full-fat milk and eggs) leads to satiety. Consuming rapidly absorbable carbohydrates leads to increased total caloric intake throughout the day (driven by insulin resistance and leptinresistance). These metabolic consequences derived from overconsuming these types of foods leads us to eat more and exercise less. In essence, eating more and exercising less doesn’t cause obesity, overconsuming rapidly absorbable carbohydrates causes us to eat more and exercise less, which then causes obesity – a subtle but important distinction.

Read the rest of the interview on MedicalResearch.comContent Not Intended as Specific Medical Advice

Page 50: MedicalResearch.com:  Medical Research Exclusive Interviews November 26 2014

For Weight Control, Focus on Healthy Foods, Not CaloriesMedicalResearch.com Interview with:

James J. DiNicolantonio, PharmDAssociate Editor BMJ Open HeartCardiovascular Research Scientist

Saint Luke’s Mid America Heart Institute

• Medical Research: What should clinicians and patients take away from your report?

• Dr. DiNicolantonio:

• 1.) Clinicians and the public health community should promote the consumptions of whole/minimally processed foods that make eating less and moving more more possible, 2.) The public health community (and patients) should stop focusing on calories, which misdirects eating away from healthy foods and towards harmful foods.

• Medical Research: What recommendations do you have for future research as a result of this study?

• Dr. DiNicolantonio: More randomized clinical trials (both ad libitum and isocaloric exchange trials) testing the metabolic effects of rapidly absorbable carbohydrates (which includes added sugars) should be performed. The literature mainly consists of short-term trials, and we do not know the full cardiometabolic consequences of overconsuming these types of refined carbohydrates over the long-term.

• Citation:

• How calorie-focused thinking about obesity and related diseases

• may mislead and harm public health. An alternative

• Sean C Lucan and James J DiNicolantonioPublic Health Nutrition: page 1 of 11

Read the rest of the interview on MedicalResearch.comContent Not Intended as Specific Medical Advice

Page 51: MedicalResearch.com:  Medical Research Exclusive Interviews November 26 2014

Three Cases of Cancer Misdiagnosed as Chronic Lyme DiseaseMedicalResearch.com Interview with:

Christina Nelson, MD, MPH, FAAP Medical EpidemiologistCenters for Disease Control and Prevention

Division of Vector-Borne Diseases | Bacterial Diseases Branch Fort Collins, CO

• Medical Research: What is the background for this study?

• Dr. Nelson: Evidence-based guidelines for the diagnosis and treatment of Lyme disease have been provided by the Infectious Diseases Society of America for many years. These comprehensive guidelines have been vetted by external review panels as the best option for patient care. In endemic areas, patients with the typical rash (erythema migrans) can be diagnosed with Lyme disease clinically. Otherwise, the guidelines recommend that diagnosis be based on a history of possible exposure, compatible clinical features, and positive two-tier serologic testing.

• Some patients who have been treated for Lyme disease may develop post-treatment Lyme disease syndrome (PTLDS) – fatigue, arthralgias, or other symptoms that persist after completing antibiotic treatment. Although the exact cause of post-treatment Lyme disease syndrome is unknown, it is thought to be due to an altered immune response or residual damage to tissues during the acute infection. A diagnosis of exclusion, PTLDS should only be diagnosed after the patient has been thoroughly evaluated and other potential causes of symptoms ruled out.

Read the rest of the interview on MedicalResearch.comContent Not Intended as Specific Medical Advice

Page 52: MedicalResearch.com:  Medical Research Exclusive Interviews November 26 2014

Three Cases of Cancer Misdiagnosed as Chronic Lyme DiseaseMedicalResearch.com Interview with:

Christina Nelson, MD, MPH, FAAP Medical EpidemiologistCenters for Disease Control and Prevention

Division of Vector-Borne Diseases | Bacterial Diseases Branch Fort Collins, CO

• On the other hand, “chronic Lyme disease” is a loosely defined diagnosis that has been used to describe a variety of ailments. A small cadre of providers use unconventional methods to diagnose patients with chronic Lyme disease, and sometimes there is no objective evidence that the patient ever had Lyme disease. Multiple factors contribute to this phenomenon, including misconceptions about serologic testing, use of unvalidated diagnostic tests, and clinical diagnosis of Lyme disease based on nonspecific symptoms alone.

• We know that patients have been – and continue to be – harmed by treatments for chronic Lyme disease. Patients have suffered from emboli, severe allergic reactions to antibiotics, neutropenia, and infections such as Clostridium difficile. This is terrible and should never happen. However, there is another important danger related to these alternative practices. Some patients who have been diagnosed and treated for chronic Lyme disease later discover that another condition is the root of their physical problem. We wanted to highlight some of these cases in order to help educate providers and patients about this issue.

Read the rest of the interview on MedicalResearch.comContent Not Intended as Specific Medical Advice

Page 53: MedicalResearch.com:  Medical Research Exclusive Interviews November 26 2014

Three Cases of Cancer Misdiagnosed as Chronic Lyme DiseaseMedicalResearch.com Interview with:

Christina Nelson, MD, MPH, FAAP Medical EpidemiologistCenters for Disease Control and Prevention

Division of Vector-Borne Diseases | Bacterial Diseases Branch Fort Collins, CO

• Medical Research: What are the main findings?

• Dr. Nelson: This paper describes three patients who had neoplasms (cancer or tumors) that caused symptoms that were incorrectly attributed to chronic Lyme disease.

• Patient 1 suffered from the effects of a pituitary tumor but was incorrectly treated for chronic Lyme disease for years. Although the tumor was eventually discovered and surgically removed, he continues to undergo treatments for the physical effects of the tumor’s unchecked growth.

• Patient 2 was treated with antibiotics for fatigue, abdominal pain, and loose stools. He was later diagnosed with Stage IV lymphoma and died two years afterward.

• Patient 3 presented with the signs of Lyme disease and was appropriately treated. Subsequently, he continued to feel unwell and received additional courses of antibiotics. A chest X-ray months later revealed lung cancer. Fortunately, he had a good outcome once the tumor was diagnosed and surgically removed. However, further delays in recognition of his actual condition would have allowed tumor growth and possible metastasis.

• Each of these patients was inappropriately treated with long-term antibiotics when their symptoms were due to another treatable condition.

Read the rest of the interview on MedicalResearch.comContent Not Intended as Specific Medical Advice

Page 54: MedicalResearch.com:  Medical Research Exclusive Interviews November 26 2014

Three Cases of Cancer Misdiagnosed as Chronic Lyme DiseaseMedicalResearch.com Interview with:

Christina Nelson, MD, MPH, FAAP Medical EpidemiologistCenters for Disease Control and Prevention

Division of Vector-Borne Diseases | Bacterial Diseases Branch Fort Collins, CO

• Medical Research: What should clinicians and patients take away from your report?

• Dr. Nelson: CDC wants people who have Lyme disease to be diagnosed and treated as quickly as possible.

• On the other hand, we don’t want people to be treated for chronic Lyme disease when they are suffering from another ailment that’s gone undiagnosed. The diagnosis of any disease should come only after a thorough evaluation and exclusion of other plausible causes. When a patient does not improve or the diagnosis no longer seems likely, the responsible provider should reassess and consider other diagnoses.

• Furthermore, the first two patients described in this report had worsening symptoms during treatment for presumed Lyme disease and were told that this was due to a Jarisch-Herxheimer reaction (commonly known as “herxing”). The Jarisch-Herxheimer reaction is an acute episode of fever, hypotension, or respiratory distress that occurs within 24 hours of initiating antibiotic treatment for some bacterial infections. There is no evidence that Jarisch-Herxheimer reactions cause ongoing or delayed worsening of symptoms. Patients who worsen during antibiotic treatment should be reevaluated thoroughly to exclude other disease processes.

• The bottom line for patients is: if you’ve been diagnosed with chronic Lyme disease or have been treated for this and do not seem to be getting better, please consider getting a second opinion to rule out other treatable conditions. You deserve an accurate diagnosis!

Read the rest of the interview on MedicalResearch.comContent Not Intended as Specific Medical Advice

Page 55: MedicalResearch.com:  Medical Research Exclusive Interviews November 26 2014

Three Cases of Cancer Misdiagnosed as Chronic Lyme DiseaseMedicalResearch.com Interview with:

Christina Nelson, MD, MPH, FAAP Medical EpidemiologistCenters for Disease Control and Prevention

Division of Vector-Borne Diseases | Bacterial Diseases Branch Fort Collins, CO

• Medical Research: What recommendations do you have for future research as a result of this study?

• Dr. Nelson: More research on post-treatment Lyme disease syndrome is needed in order to better understand the pathogenesis of this condition and identify effective treatment options for patients. This is a difficult condition that continues to cause patient morbidity.

• Also, it would be helpful to have systematic assessments of misdiagnosis and harmful treatments for chronic Lyme disease so that we can get a better handle on the magnitude of this problem. Patients continue to be harmed by unconventional approaches to diagnosis and treatment of Lyme disease. This report and other published cases of adverse events are unfortunately only the tip of the iceberg.

• Citation:

• Nelson C, Elmendorf S, Mead P. Neoplasms Misdiagnosed as “Chronic Lyme Disease”. JAMA Intern Med. Published online November 03, 2014. doi:10.1001/jamainternmed.2014.5426.

Read the rest of the interview on MedicalResearch.comContent Not Intended as Specific Medical Advice

Page 56: MedicalResearch.com:  Medical Research Exclusive Interviews November 26 2014

Bleeding and Ischemic Events Following Bare Metal vs Drug Eluting StentsMedicalResearch.com Interview with:

Dean J. Kereiakes, MD FACC, FSCAIThe Lindner Research Center The Christ Hospital Health Network

Cincinnati, Ohio 45219

• Medical Research: What is the background for this study?

• Dr. Kereiakes: Bare metal stents (BMS) are a commonly used alternative to drug eluting stents (DES) particularly for patients presenting with acute coronary syndromes or in whom dual antiplatelet therapy (DAPT) has perceived increased bleeding risks. We aimed to determine whether the risks of stent thrombosis and major adverse clinical cardiovascular and cerebrovascular (MACCE; composite of death, MI or stroke) events differ for BMS versus DES and whether the optimal duration of dual antiplatelet therapy differs for BMS or DES. To answer these objectives we performed a propensity matched BMS to DES 0-33 month comparison as well as an analysis of treatment effect among BMS treated patients randomly assigned to 12 versus 30 months of DAPT.

Read the rest of the interview on MedicalResearch.comContent Not Intended as Specific Medical Advice

Page 57: MedicalResearch.com:  Medical Research Exclusive Interviews November 26 2014

Bleeding and Ischemic Events Following Bare Metal vs Drug Eluting StentsMedicalResearch.com Interview with:

Dean J. Kereiakes, MD FACC, FSCAIThe Lindner Research Center The Christ Hospital Health Network

Cincinnati, Ohio 45219

• Medical Research: What are the main findings?

• Dr. Kereiakes: In 10,026 DES and BMS treated subjects matched exactly on the diagnosis of STEMI and for all remaining (total 55) variables via propensity score using a caliper 0.10, DES were observed to have a significantly lower incidence of stent thrombosis (Academic Research Consortium [ARC] definite/probable definition) from 0 -33 months with a trend (p=0.052) toward a lower incidence of MACCE as well. Among 1687 BMS-treated patients randomly assigned to 30 versus 12 DAPT, stent thrombosis was reduced by longer duration therapy (stratified hazard ratio 0.49; stratified log rank p value 0.24). The 51% reduction in stent thrombosis by longer duration therapy was entirely explained by a reduction in the occurrence of definite stent thrombosis (ARC definition). Bleeding events were increased with longer (30 months) duration DAPT (GUSTO severe- moderate bleeding 2.03% versus 0.9% with shorter therapy; p=0.07). Finally, no difference in mortality (cardiovascular or non-cardiovascular) was observed between the 30 and 12 month treatment groups.

Read the rest of the interview on MedicalResearch.comContent Not Intended as Specific Medical Advice

Page 58: MedicalResearch.com:  Medical Research Exclusive Interviews November 26 2014

Bleeding and Ischemic Events Following Bare Metal vs Drug Eluting StentsMedicalResearch.com Interview with:

Dean J. Kereiakes, MD FACC, FSCAIThe Lindner Research Center The Christ Hospital Health Network

Cincinnati, Ohio 45219

• Medical Research: What should clinicians and patients take away from your report?

• Dr. Kereiakes: Clinicians and patients should take away the following:

• BMS are not safer than DES. In fact, BMS have a higher risk for stent thrombosis than DES.

• The treatment effect benefit of longer (30 months) DAPT compared with shorter (12 months) therapy appears consistent for both DES and BMS. This similarity in treatment effect benefit is reflected by hazard ratio reductions of <1.0 (0.49 for BMS; 0.29 for DES) as well as no statistically significant evidence for treatment duration by stent type interaction (p=0.42).

• Bleeding risk is increased with longer duration DAPT but fatal bleeding was rare and not different between randomized treatment groups (30 versus 12 months).

• No differences in mortality were evident between subjects randomly assigned to 30 versus 12 months of DAPT following BMS deployment.

Read the rest of the interview on MedicalResearch.comContent Not Intended as Specific Medical Advice

Page 59: MedicalResearch.com:  Medical Research Exclusive Interviews November 26 2014

Bleeding and Ischemic Events Following Bare Metal vs Drug Eluting StentsMedicalResearch.com Interview with:

Dean J. Kereiakes, MD FACC, FSCAIThe Lindner Research Center The Christ Hospital Health Network

Cincinnati, Ohio 45219

• Medical Research: What recommendations do you have for future research as a result of this study?

• Dr. Kereiakes: These results will likely inform clinical practice guidelines as well as clinical practice. Despite the widely held perception that BMS are “safer” and require a shorter duration of DAPT than DES, our results demonstrate that BMS have a higher risk for stent thrombosis and derive treatment benefit from longer duration DAPT (30 months versus 12 months) which is consistent with that observed for DES. In BMS treated patients, prolonger thienopyridine (30 months or longer) may provide durable ischemic benefit to increased bleeding risk and requires further study.

• Citation:

• AHA14 abstract:

• Comparison of Ischemic and Bleeding Events After Drug-Eluting Stents or Bare Metal Stents in Subjects Receiving Dual Antiplatelet Therapy: Results from the Randomized Dual Antiplatelet Therapy Study

Read the rest of the interview on MedicalResearch.comContent Not Intended as Specific Medical Advice

Page 60: MedicalResearch.com:  Medical Research Exclusive Interviews November 26 2014

Physicans and Nurses Traumatized By Medical ErrorsMedicalResearch.com Interview with :

Alexandra LaurentMaître de conférences de psychologie clinique et psychopathologieLaboratoire de psychologie EA3188 Université de Franche-Comté

• Medical Research : What is the background for this study? What are the main findings?

• Dr. Laurent: Human error among healthcare professionals is a subject of current affairs and especially in ICUs which are among the services with a high risk of error. If the error affects the patient and his/her family, it will also have an impact on the caregivers involved, their colleagues, and even the entire service. In an editorial in the BMJ, Wu introduced the term “the second victim” to define a caregiver implicated in and traumatised by an medical error for which he/she feels personally responsible. Therefore, it’s important to improve understanding of the psychological repercussions of error on professionals in ICUs, and to identify the defense mechanisms used by professionals to cope with errors.

• In the month following the error, We found that the professionals (doctors and nurses) described feelings of guilt and shame. These feelings were associated with: anxiety states with rumination and fear for the patient; a loss of confidence; an inability to verbalize one’s error; questioning oneself at a professional level; and anger towards the team.

• In the long term, the error remains fixed in memory with, on one hand, for 72.5%, an increase in vigilance and verifications in their professional practice and, on the other hand, for 3 professionals, a loss of confidence. Finally, 3 professionals felt guilt which still persisted at the time of the interview.

Read the rest of the interview on MedicalResearch.comContent Not Intended as Specific Medical Advice

Page 61: MedicalResearch.com:  Medical Research Exclusive Interviews November 26 2014

Physicans and Nurses Traumatized By Medical ErrorsMedicalResearch.com Interview with :

Alexandra LaurentMaître de conférences de psychologie clinique et psychopathologieLaboratoire de psychologie EA3188 Université de Franche-Comté

• We also observed different defense mechanisms implemented by the professional in order to fight against the emotional load inherent in the error: verbalization, developing skills and knowledge, rejecting responsibility, and avoidance. We also observed a minimization of the error during the interviews.

• Finally, our study shows that is important to take into account the psychological experience of error and the defense mechanisms developed following an error because they appear to determine the professional’s capacity to acknowledge and disclose his/her error, and to learn from it.

• Medical Research: What should clinicians take away from your report?

• Dr. Laurent: The challenge for developing a policy of security of care is: the need to verbalize errors in order to learn from them. However, before encouraging the ‘disclosure of error’, one must take into account the professional’s experience and the way in which he/she copes with it. Our results show that the effects of shame and guilt felt by the professionals following an error impact the ability of the subject to disclose his/her error and, by doing so, to receive the support of the group to cope with the event. We observed that the effects of shame and guilt were accompanied by defence mechanisms which served to reduce the emotional load and enabled the professional to continue working. On the other hand, these same defence mechanisms hinder the subject’s ability to acknowledge the error and to learn from it.

Read the rest of the interview on MedicalResearch.comContent Not Intended as Specific Medical Advice

Page 62: MedicalResearch.com:  Medical Research Exclusive Interviews November 26 2014

Physicans and Nurses Traumatized By Medical ErrorsMedicalResearch.com Interview with :

Alexandra LaurentMaître de conférences de psychologie clinique et psychopathologieLaboratoire de psychologie EA3188 Université de Franche-Comté

• Medical Research: What recommendations do you have for future research as a result of this study?

• Dr. Laurent: The MMRs (Mortality and Morbidity Reviews) and/or the creation of a space for discussion within the services such as debriefing could facilitate the disclosure of error, we must advance with caution. Our research shows that there are dangers in insisting that the professional talk about his/her error if the service has not installed a sufficiently reassuring and comprehensive work climate beforehand. The seniors and heads of departments have an important role to play in this climate of confidence because it is essential that the error be discussed without fear and without fear of judgement, a reprimand or professional disqualification. However, this does not imply minimising the caregiver’s responsibility for an error. On the contrary, support in the face of error serves to restore the caregiver’s self-confidence and facilitates discussion of the repercussions of the error in order to learn from it.

• Thus, an ICU culture that encourages caregivers to acknowledge and take responsibility for their errors empowers them in situations that can initially be perceived as hopeless and out of control.

• Citation :

• Laurent A., Aubert L., Chahraoui K., Bioy A., Mariage A., Quenot JP., Capellier G. (2014). The error in intensive care: psychological repercussions and defense mechanisms. Critical Care Medicine, 42, 11, 2370-2378

Read the rest of the interview on MedicalResearch.comContent Not Intended as Specific Medical Advice

Page 63: MedicalResearch.com:  Medical Research Exclusive Interviews November 26 2014

Some Diabetics Do Not Benefit From ExerciseMedicalResearch.com Interview with:

Lauren Marie Sparks, PhDFaculty Scientist at the Translational Research Institute for Metabolism and Diabetes

Florida Hospital and the Sanford-Burnham Medical Research Institute Orlando, FL

• Medical Research: What is the background for this study? What are the main findings?

• Dr. Sparks: As a clinical scientist focused on exercise effects on muscle metabolism in diabetes, I have seen first-hand a significant minority of individuals with diabetes not improve their glucose control (HbA1c) after 9 months of supervised exercise. They poured their hearts out on those treadmills 3-4 days a week for 9 months and still ended up no better than when they started. I have also seen similar data from some of my colleagues’ studies. So I really want diabetes research to invest the intellect and dollars into discovering what these roadblocks are—I happen to believe it is in the DNA (genetics) and the way that DNA is “read” or expressed (epigenetics). So it’s a bit of a ‘call to action’ for researchers to start looking into some of their data to find these people and better understand this phenomenon and for hopefully the funding sources to recognize this as a viable area of research.

Read the rest of the interview on MedicalResearch.comContent Not Intended as Specific Medical Advice

Page 64: MedicalResearch.com:  Medical Research Exclusive Interviews November 26 2014

Some Diabetics Do Not Benefit From ExerciseMedicalResearch.com Interview with:

Lauren Marie Sparks, PhDFaculty Scientist at the Translational Research Institute for Metabolism and Diabetes

Florida Hospital and the Sanford-Burnham Medical Research Institute Orlando, FL

• Medical Research: What should clinicians and patients take away from your report?

• Dr. Sparks: On the clinical side, I want practitioners to understand that some people really don’t respond to exercise—and it’s not for lack of trying. So a lack of a positive metabolic result (for diabetics, usually measured by a decrease in their HbA1c) doesn’t always mean a lack of effort. And hopefully in time the science will catch up. Right now, we know these people exist—so called “non-responders”—but we don’t currently have strategies to appropriately help them in their journey toward a healthier self. Take home is message is for clinicians to still prescribe exercise to the patients with diabetes because exercise is a full body experience that will certainly have lots of beneficial effects—such as quality of life, etc. And for those that simply are not responding, don’t tell them to stop exercising, but just continue to work with them. The overall goal of the research in this area is to identify these ‘non-responders’ to exercise and begin to develop novel strategies to treat them—i.e. imagine one day that a person could take a pill that “releases the brake” on their DNA and allows them to respond to the exercise that they are doing.

Read the rest of the interview on MedicalResearch.comContent Not Intended as Specific Medical Advice

Page 65: MedicalResearch.com:  Medical Research Exclusive Interviews November 26 2014

Some Diabetics Do Not Benefit From ExerciseMedicalResearch.com Interview with:

Lauren Marie Sparks, PhDFaculty Scientist at the Translational Research Institute for Metabolism and Diabetes

Florida Hospital and the Sanford-Burnham Medical Research Institute Orlando, FL

• Medical Research: What recommendations do you have for future research as a result of this study?

• Dr. Sparks: Clinically, the recommendation is for patients with T2Diabetes to STILL EXERCISE. We are not close to having all of the answers on this topic just yet. And exercise is a full body experience that still has many beneficial effects beyond what we have looked at here. Also, we don’t want people to start ‘self-diagnosing’ as “exercise resistant”.

• This publication and research is a bit of a ‘call to action’ for researchers to start looking into some of their data to find these people and better understand this phenomenon and for hopefully the funding sources to recognize this as a viable area of research. I am also halfway through a study funded by the ADA to look at this exact phenomenon—so the hope is that in the next few years, we will have more concrete answers. Imagine that one day a person could take a pill prior to training in order to “release this brake” on their DNA and benefit from the exercise in which they are engaging.

• Citation:

• Natalie A. Stephens, Lauren M. Sparks. Resistance to the Beneficial Effects of Exercise in Type 2 Diabetes: Are Some Individuals Programmed to Fail? The Journal of Clinical Endocrinology & Metabolism, 2014; jc.2014-2545 DOI: 10.1210/jc.2014-2545

Read the rest of the interview on MedicalResearch.comContent Not Intended as Specific Medical Advice

Page 66: MedicalResearch.com:  Medical Research Exclusive Interviews November 26 2014

Reduced Sodium Intake May Improve Heart Failure PrognosisMedicalResearch.com Interview with:

Dr. Eloisa Colin-Ramirez, BSc, PhD and Justin A. Ezekowitz, MBBCh MScAssociate Professor, University of Alberta

Co-Director, Canadian VIGOUR Centre Director, Heart Function Clinic

• Medical Research: What is the background for this study? What are the main findings?

• Response: The SODIUM-HF study is a randomized control trial on sodium restriction in patients with chronic heart failure (HF). Sodium restriction has been broadly recommended as part of the self-care strategies in heart failure yet is based on little high-quality evidence. This study reports the results of the pilot SODIUM-HF trial in 38 patients with chronic HF. Nineteen patients were prescribed a low sodium containing diet (1500 mg/day) and 19 a moderate sodium containing diet (2300 mg/day). Both interventions were based on a structured and individualized meal plan to achieve the targeted sodium intake, and all patients were followed for 6 months with monthly phone call to reinforce adherence to the diet.

• We found a meaningful reduction in sodium intake to less than 1500 mg/day at 6 months in both groups. Additionally, we observed that patients that achieved a sodium intake less than 1500 mg/day at 6 months of follow-up had reduced BNP levels, a biomarker of volume overload and surrogate prognostic marker in heart failure, and increased overall and clinical scores of the Kansas City Cardiomyopathy Questionnaire, compared to those with a sodium intake greater than 1500 mg/day.

Read the rest of the interview on MedicalResearch.comContent Not Intended as Specific Medical Advice

Page 67: MedicalResearch.com:  Medical Research Exclusive Interviews November 26 2014

Reduced Sodium Intake May Improve Heart Failure PrognosisMedicalResearch.com Interview with:

Dr. Eloisa Colin-Ramirez, BSc, PhD and Justin A. Ezekowitz, MBBCh MScAssociate Professor, University of Alberta

Co-Director, Canadian VIGOUR Centre Director, Heart Function Clinic

• Medical Research: What recommendations do you have for future research as a result of this study?

• Response: Currently, the SODIUM-HF trial (SODIUM-HF; clinicaltrials.gov NCT02012179) is being conducted. In order to ensure the intervention leads to a more meaningful differences in sodium intake between groups, this multicentre trial is employing the same dietary strategies used in this pilot to counsel patients in the low-sodium diet group; however, the Usual Care group is receiving usual dietary recommendation for sodium restriction and follow-up as provided during routine clinical practice and supported by the Canadian heart failure guidelines. This ongoing multicenter trial is expected to provide definite results on the effects of sodium restriction in HF patients and develop evidence-based guideline for sodium restriction in this patient population.

• Citation:

• The long-term effects of dietary sodium restriction on clinical outcomes in patients with heart failure. The SODIUM-HF (Study of Dietary Intervention Under 100 MMOL in Heart Failure): A Pilot Study

• Colin-Ramirez, Eloisa et al. American Heart Journal November 11, 2014; Published Online: November 19, 2014

• DOI: http://dx.doi.org/10.1016/j.ahj.2014.11.013

Read the rest of the interview on MedicalResearch.comContent Not Intended as Specific Medical Advice

Page 68: MedicalResearch.com:  Medical Research Exclusive Interviews November 26 2014

Reduced Sodium Intake May Improve Heart Failure PrognosisMedicalResearch.com Interview with:

Dr. Eloisa Colin-Ramirez, BSc, PhD and Justin A. Ezekowitz, MBBCh MScAssociate Professor, University of Alberta

Co-Director, Canadian VIGOUR Centre Director, Heart Function Clinic

• Medical Research: What should clinicians and patients take away from your report?

• Response:

• First, results of this pilot study suggest that dietary sodium reduction in heart failure is feasible when an individualized and structured meal plan with close follow-up is provided to patients.

• Secondly, an achieved sodium intake less than 1500 mg/day at 6 months of follow-up was associated with reduced BNP levels and improved quality of life in ambulatory patients with heart failure on optimal medical treatment. However, larger RCTs with clinical outcomes as primary endpoints are required to support this recommendation and confirm a better prognosis associated to less sodium intake in patients with HF.

Read the rest of the interview on MedicalResearch.comContent Not Intended as Specific Medical Advice

Page 69: MedicalResearch.com:  Medical Research Exclusive Interviews November 26 2014

Study Addresses Nutritional Support Of Critically Ill PatientsMedicalResearch.com Interview with:

Sheila E. Harvey, Ph.D.CTU Manager/Senior Research Fellow ICNARC

Napier House London

• Medical Research: What is the background for this study?

• Dr. Harvey: The CALORIES trial was set-up in the context of concerns about malnutrition in critically ill patients in NHS hospitals and conflicting evidence as to the optimal route for delivery of early nutritional support to critically ill patients. The enteral route is the mainstay of nutritional support in the critically ill but it is frequently associated with gastrointestinal intolerance and underfeeding. In contrast, the parenteral route, though more invasive and expensive, is more likely to secure delivery of the intended nutrition but has been associated with more risks and complications (e.g. infectious complications) compared with the enteral route.

• In light of the uncertainty surrounding the most effective route for delivery of early nutritional support and, given recent improvements in the delivery, formulation and monitoring of parenteral nutrition, the UK National Institute of Health Research (NIHR) Health Technology Assessment (HTA) Programme put out a “call” for a large pragmatic randomised controlled trial to be conducted in critically ill patients to determine the optimal route of delivery of early nutritional support. CALORIES was set up to test the hypothesis that early nutritional support delivered via the parenteral route is superior to early nutritional support delivered via the enteral route in adults who had an unplanned admission to an intensive care unit and who could be fed via either route.

• The primary outcome was all-cause mortality at 30 days. The secondary outcomes included infectious and non-infectious complications (hypoglycaemia, elevated liver enzymes, nausea requiring treatment, abdominal distension, vomiting, new or substantially worsened pressure ulcers).

Read the rest of the interview on MedicalResearch.comContent Not Intended as Specific Medical Advice

Page 70: MedicalResearch.com:  Medical Research Exclusive Interviews November 26 2014

Study Addresses Nutritional Support Of Critically Ill PatientsMedicalResearch.com Interview with:

Sheila E. Harvey, Ph.D.CTU Manager/Senior Research Fellow ICNARC

Napier House London

• Medical Research: What are the main findings?

• Dr. Harvey: Caloric intake was similar in the two groups, however, the target of 25 kcal/kg/day was not met in most patients. All-cause 30-day mortality was similar for patients allocated to receive early nutritional support via the parenteral route (33.1%) and via the enteral route (34.2%). The rates of hypoglycaemia and vomiting were significantly higher in the enteral route group. However, there was no significant difference between the groups in the mean number of treated infectious complications, or in any of the other secondary outcomes or in the rate of adverse events.

• Medical Research: What should clinicians and patients take away from your report?

• Dr. Harvey: The CALORIES trial evaluated delivery of early nutritional support via the parenteral compared with the enteral route, as typically administered in critical care units in NHS hospitals in England. The trial has demonstrated that previous beliefs about the risks of feeding patients via the parenteral route (e.g. infectious complications) have been shown not to be true in the present day in English intensive care units. In addition, the assumption that early nutritional support delivered via the parenteral route automatically guarantees delivery of the intended nutrition was also shown not to be true. Longer-term follow-up of patients is ongoing and we await the cost-effectiveness analysis. The CALORIES trial did not address the question of nutritional dose, however the findings suggest that the choice of route for nutritional support can be determined by clinical need without concerns about potential major direct harm from either route.

Read the rest of the interview on MedicalResearch.comContent Not Intended as Specific Medical Advice

Page 71: MedicalResearch.com:  Medical Research Exclusive Interviews November 26 2014

Study Addresses Nutritional Support Of Critically Ill PatientsMedicalResearch.com Interview with:

Sheila E. Harvey, Ph.D.CTU Manager/Senior Research Fellow ICNARC

Napier House London

• Medical Research: What recommendations do you have for future research as a result of this study?

• Dr. Harvey: The findings of the CALORIES trial suggest that early nutritional support via the parenteral route is neither more harmful nor more beneficial than early nutritional support via the enteral route. The question as to what is the optimal nutritional dose and the optimal method for determination of energy and protein or amino acid requirements for critically ill patients are important future research questions in this field.

• Citation:

• Trial of the Route of Early Nutritional Support in Critically Ill AdultsSheila E. Harvey, Ph.D., Francesca Parrott, M.Sci., David A. Harrison, Ph.D., Danielle E. Bear, M.Res., Ella Segaran, M.Sc., Richard Beale, M.B., B.S., Geoff Bellingan, M.D., Richard Leonard, M.B., B.Chir., Michael G. Mythen, M.D., and Kathryn M. Rowan, Ph.D. for the CALORIES Trial Investigators

• N Engl J Med 2014; 371:1673-1684October 30, 2014DOI: 10.1056/NEJMoa1409860

Read the rest of the interview on MedicalResearch.comContent Not Intended as Specific Medical Advice

Page 72: MedicalResearch.com:  Medical Research Exclusive Interviews November 26 2014

Drop in Cardiac Mortality Varies by Subtype and EthnicityMedicalResearch.com Interview with:

Matthew D. Ritchey, DPTDivision for Heart Disease and Stroke Prevention

US Centers for Disease Control and Prevention, Atlanta, Georgia

• Medical Research: What is the background for this study? What are the main findings?

• Dr. Ritchey: This study analyzes the contribution of heart disease subtypes – such as coronary heart disease, heart failure, hypertensive heart disease and arrhythmia – to overall trends in heart disease death rates between 2000 and 2010. Our research revealed that overall heart disease-related deaths declined during that time frame at a rate of almost four percent annually. Most of this decline appears to be driven by decreases in coronary heart disease mortality, which includes deaths due to heart attacks.

• However, not all heart disease subtypes saw similar decreases. Arrhythmia and hypertensive heart disease death rates increased annually during this period. In addition, there were differences depending on age group, subtype, gender and race/ethnicity. For example, hypertensive heart disease rates were much higher (more than double) among non-Hispanic blacks in 2010 than among non-Hispanic whites. That could be due to factors including uncontrolled blood pressure and obesity among younger adults. Also, the increase in arrhythmia mortality was highest among non-Hispanic whites, women and adults age 75 and over. That increase might be linked to the growing aging population, the result of individuals living longer with heart failure, increases in chronic kidney disease and hypertensive heart disease prevalence and changes in how the condition is reported.

• To determine these findings, we examined de-identified death certificates of U.S. residents ages 35 and up who died from 2000 to 2010. The data was pulled from the CDC WONDER database, which contains death certificate information from every U.S. state and the District of Columbia.

Read the rest of the interview on MedicalResearch.comContent Not Intended as Specific Medical Advice

Page 73: MedicalResearch.com:  Medical Research Exclusive Interviews November 26 2014

Drop in Cardiac Mortality Varies by Subtype and EthnicityMedicalResearch.com Interview with:

Matthew D. Ritchey, DPTDivision for Heart Disease and Stroke Prevention

US Centers for Disease Control and Prevention, Atlanta, Georgia

• Medical Research: What should clinicians and patients take away from your report?

• Dr. Ritchey: Between 2000 and 2010, more than 7 million heart disease deaths occurred, including 600,000 deaths in 2010. These statistics show that we still have work to do to create a heart-healthy world.

• Heart disease remains one of the most widespread and costly health problems facing the nation, so this research is beneficial because it helps reveal disparities and existing and potential trends. Knowing how the risks break down – including by race, gender and heart disease subtype – will help clinicians, patients and public health representatives focus efforts as needed. Clinicians should continue to develop and apply evidence-based interventions to prevent and treat all subtypes of heart disease, such as those stressed by Million Hearts®, a national initiative aimed at preventing one million heart attacks and strokes by 2017. Patients can use this information as incentive to make lifestyle changes aimed at reducing their own cardiovascular risk, including quitting smoking and managing blood pressure and cholesterol.

Read the rest of the interview on MedicalResearch.comContent Not Intended as Specific Medical Advice

Page 74: MedicalResearch.com:  Medical Research Exclusive Interviews November 26 2014

Drop in Cardiac Mortality Varies by Subtype and EthnicityMedicalResearch.com Interview with:

Matthew D. Ritchey, DPTDivision for Heart Disease and Stroke Prevention

US Centers for Disease Control and Prevention, Atlanta, Georgia

• Medical Research: What recommendations do you have for future research as a result of this study?

• Dr. Ritchey: We will continue to track overall heart disease mortality trends, as well as trends by heart disease subtype, to help guide heart disease prevention efforts nationwide. In addition, further research should be considered to examine what factors are causing increases in hypertensive heart disease and arrhythmia ratesCitation:

• Ritchey MD, Loustalot F, Bowman BA, Hong Y. Trends in Mortality Rates by Subtypes of Heart Disease in the United States, 2000-2010. JAMA. 2014;312(19):2037-2039. doi:10.1001/jama.2014.11344.

Read the rest of the interview on MedicalResearch.comContent Not Intended as Specific Medical Advice

Page 75: MedicalResearch.com:  Medical Research Exclusive Interviews November 26 2014

Obesity in Midlife Increases Risk of Dementia and Alzheimer’s DiseaseMedicalResearch.com Interview with:

Nicolas Cherbuin PhDARC Future Fellow – Director of the NeuroImaging and Brain Lab Centre for Research on Ageing, Health and Wellbeing

Research School of Population Health – College of Medicine Biology and EnvironmentAustralian National University

• This study investigated 420 participants in their early 60s taking part in a larger longitudinal study of aging taking place in Canberra, Australia and who underwent up to three brain scans over an 8-year follow-up. These individuals were free of dementia and other neurological disorders. Associations between obesity and shrinkage of the hippocampus were investigated with longitudinal analyses which controlled for major confounders.

• The main findings were that overweight and obese participants had smaller volume of the hippocampus at the start of the study. In addition, the hippocampus shrunk more in these individuals over the follow-up period.

• Medical Research: What should clinicians and patients take away from your report?

• Dr. Cherbuin: Obesity is a significant risk factor for cerebral health and our findings suggest that these effects are not diminished in aging.

• Medical Research: What recommendations do you have for future research as a result of this study?

• Dr. Cherbuin: Further research should be conducted to determine the extent to which obesity is associated with cognitive decline across the lifespan. The biological mechanisms underlying these effects should be investigated in details. Developing effective interventions to change behaviour, reduce energy intake, discourage consumption of processed and fast foods and soft drinks, and incite greater physical activity should be a priority.

Read the rest of the interview on MedicalResearch.comContent Not Intended as Specific Medical Advice

Page 76: MedicalResearch.com:  Medical Research Exclusive Interviews November 26 2014

Obesity in Midlife Increases Risk of Dementia and Alzheimer’s DiseaseMedicalResearch.com Interview with:

Nicolas Cherbuin PhDARC Future Fellow – Director of the NeuroImaging and Brain Lab Centre for Research on Ageing, Health and Wellbeing

Research School of Population Health – College of Medicine Biology and EnvironmentAustralian National University

• Medical Research: Is the hippocampus shrinkage reversible?

• Dr. Cherbuin: At this stage it is not completely clear whether some shrinkage is reversible. The hippocampus is one of two brain regions where significant numbers of new neurons are produced. Given obesity is associated with a slowing of production of new neurons (neurogenesis) in animals it would be expected that weight loss would be associated with some recovery.

• One animal study (Grayson et al. 2014) has shown that obese animals that were administered two types of gastric surgery performed similarly to control rats with normal weight on a caloric restriction diet whereas untreated obese rats performed significantly worse on a memory task. This suggests that at least recovery in function is possible in animals.

Read the rest of the interview on MedicalResearch.comContent Not Intended as Specific Medical Advice

Page 77: MedicalResearch.com:  Medical Research Exclusive Interviews November 26 2014

Obesity in Midlife Increases Risk of Dementia and Alzheimer’s DiseaseMedicalResearch.com Interview with:

Nicolas Cherbuin PhDARC Future Fellow – Director of the NeuroImaging and Brain Lab Centre for Research on Ageing, Health and Wellbeing

Research School of Population Health – College of Medicine Biology and EnvironmentAustralian National University

• Medical Research: Based on your results, what advice would you give to those who are worried about cognitive decline?

• Dr. Cherbuin: We should all be concerned by cognitive decline. The way to decrease our anxiety about it is to lead healthier lives which includes eating better food (little or no fast food and no soft drinks, plenty of fish, little meat, and plenty of fruit and vegetables) in smaller quantities, keeping track of our weight and remaining in the healthy weight range (there is no biological reason to become heavier with age), exercise (2-3 times a week at medium/high intensities throughout life; walk or cycle to work; walk up the stairs, work standing when possible, etc), monitor health and seek health advice regarding chronic conditions, deal with mental health (there is a reciprocal ling between mood disorders, obesity, cardiometabolic disease and cerebral health and dementia).

• Citation:

• Neuroscience14 abstract:

Obesity and being overweight is associated with hippocampal atrophy: the path through life study

Read the rest of the interview on MedicalResearch.comContent Not Intended as Specific Medical Advice

Page 78: MedicalResearch.com:  Medical Research Exclusive Interviews November 26 2014

First Episode of Psychosis: Stopping Drug Use Improves PrognosisPosted on November 26, 2014

MedicalResearch.com Interview with:Melissa Anne Elin Authen Weibell

Consultant PsychiatristHelse Stavanger HF

• Medical Research: What is the background for this study?

• Dr. Weibell: Little is known about the effect of different patterns of substance use on outcomes in first-episode psychosis and the few studies that exist are often cross-sectional and heterogeneous. This new study investigated different patterns of substance use in an epidemiological first-episode psychosis (FEP) sample longitudinally, with the hypothesis that continuous use would predict poorer outcomes compared to never users or stop users.

• The study included 301 patients aged 16-65 with first episode non-affective psychosisincluded (1997-2001) from three separate catchment areas in Norway and Denmark. Four patterns of substance use were defined; never used (153 patients), persistent use(43), completely stopped use having previously used (36), and on-off use (48) during the first 2-years of follow-up. 184 patients were followed up at 10 years and compared on symptom levels and remission status.

Read the rest of the interview on MedicalResearch.comContent Not Intended as Specific Medical Advice

Page 79: MedicalResearch.com:  Medical Research Exclusive Interviews November 26 2014

First Episode of Psychosis: Stopping Drug Use Improves PrognosisPosted on November 26, 2014

MedicalResearch.com Interview with:Melissa Anne Elin Authen Weibell

Consultant PsychiatristHelse Stavanger HF

• Medical Research: What are the main findings?

Dr. Weibell: The researchers found patients who stopped using within the first two years of entering treatment had similar 10-year outcomes as patients who had never used with significantly lower symptom levels (including depressive symptoms) compared to patients with on-off or persistent use.

• There was a trend for persistent users showing increasing negative symptoms over time. Importantly, there was a large and significant difference in remission rates, with 57% of never users and 63% of ‘stop’ users achieving remission at 10 years compared to 32% for on-off users and 34% for persistent users.

Read the rest of the interview on MedicalResearch.comContent Not Intended as Specific Medical Advice

Page 80: MedicalResearch.com:  Medical Research Exclusive Interviews November 26 2014

First Episode of Psychosis: Stopping Drug Use Improves PrognosisPosted on November 26, 2014

MedicalResearch.com Interview with:Melissa Anne Elin Authen Weibell

Consultant PsychiatristHelse Stavanger HF

• Medical Research: What should clinicians and patients take away from your report?

Dr. Weibell: Our results show that substance use cessation in first-episode psychosis is associated with similar outcomes to first-episode psychosis patients who never used any substances; on-off use may be almost as detrimental to mental health as persistent use. The harmful effects of substance use in first-episode psychosis can be substantially reduced if clinicians are able to assist patients to stop using altogether early on in the course of treatment.

• There are few longitudinal studies with long-term follow-up of first-episode psychosis patients with regards to substance use, so this study is really exciting and has a strong and easy to follow clinical take-home message for first episode psychosis patients: “stop using drugs early on and you will probably do better in the long-term”. For clinicians, we need to focus on stopping use as early as possible and probably at the same time that we are treating the initial psychosis. This study also presents a strong argument against the legalisation of cannabis.

Read the rest of the interview on MedicalResearch.comContent Not Intended as Specific Medical Advice

Page 81: MedicalResearch.com:  Medical Research Exclusive Interviews November 26 2014

First Episode of Psychosis: Stopping Drug Use Improves PrognosisPosted on November 26, 2014

MedicalResearch.com Interview with:Melissa Anne Elin Authen Weibell

Consultant PsychiatristHelse Stavanger HF

• Medical Research: What recommendations do you have for future research as a result of this study?

• Dr. Weibell: There is little evidence currently that any specific treatment programs can bring about sustained abstinence among patients with psychosis. Reducing substance use among FEP patients could improve the course of psychosis and treatment of substance use should be viewed as a key element of treatment programs, alongside initiation and maintenance of antipsychotic medication and psychosocial programs. The development of effective treatments for substance use in FEP patients should remain a focus and priority for further research.

• Citation:upcoming International Early Psychosis Conference abstract discussing:

• People who continue illegal substance use are much less likely to recover from first-episode psychosis

Read the rest of the interview on MedicalResearch.comContent Not Intended as Specific Medical Advice

Page 82: MedicalResearch.com:  Medical Research Exclusive Interviews November 26 2014

Guidelines For Surgery After Cardiac Stents Have Improved Adverse Postoperative OutcomesMedicalResearch.com Interview with Dr. Mary T. Hawn MD

Center for Surgical, Medical Acute Care Research, and Transitions,Birmingham Veterans Affairs Medical Center

University of Alabama at Birmingham, Birmingham

• Medical Research: What are the main findings?

• Dr. Hawn: The main findings of the study are that the recommendations made in the guidelines published by the American College of Cardiology / American Heart Association in 2007 were effective at reducing postoperative major adverse cardiac events following noncardiac surgery in patients with a cardiac stent.1 These guidelines recommended the delay of noncardiac surgeries in patients with a drug-eluting stent for 365 days if the surgery was not emergent or the delay of surgery for 4 to 6 weeks among patients with a bare metal stent. In addition to a 26% reduction in postoperative major adverse cardiac events, we also found an increase in the time between drug-eluting stent placement and non-cardiac surgery consistent with the guideline recommendations.

Medical Research: Were any of the findings unexpected?

• Dr. Hawn: In addition to our primary finding of a reduction in postoperative major adverse cardiac events, we also found evidence of a shift in cardiac risk to patients with a bare metal stent for surgeries occurring between 2006 and 2009. Previous studies have suggested that differences in the outcomes of patients with a bare metal stent as compared to a drug-eluting stent are due to confounding by indication, where sicker patients needing more urgent operations were more likely to receive a bare metal stent and also more likely to experience adverse postoperative events. This study provides evidence of a shift toward higher cardiac risk in patients with a bare metal stent following the initial publication in 2006 of several case reports of postoperative stent thrombosis in patients with a drug-eluting stent.

Read the rest of the interview on MedicalResearch.comContent Not Intended as Specific Medical Advice

Page 83: MedicalResearch.com:  Medical Research Exclusive Interviews November 26 2014

Guidelines For Surgery After Cardiac Stents Have Improved Adverse Postoperative OutcomesMedicalResearch.com Interview with Dr. Mary T. Hawn MD

Center for Surgical, Medical Acute Care Research, and Transitions,Birmingham Veterans Affairs Medical Center

University of Alabama at Birmingham, Birmingham

• Medical Research: What should clinicians and patients take away from your report?

• Dr. Hawn: While the recommendations were effective at increasing time between drug-eluting stent placement and surgery as well as reducing major adverse cardiac events, the trends in timing of surgery following drug-eluting stent placement suggest a high level of awareness among healthcare professionals treating patients with cardiac stents as well as an increased use of evidence-based practices. This resulted in an increase in surgery timing for drug-eluting stent patients which can be seen prior to the publication of the guidelines. While the publication of the guidelines marks a summary of this information into one document, it does not necessarily mark the time at which the information was officially available to healthcare providers in the field.

Read the rest of the interview on MedicalResearch.comContent Not Intended as Specific Medical Advice

Page 84: MedicalResearch.com:  Medical Research Exclusive Interviews November 26 2014

Guidelines For Surgery After Cardiac Stents Have Improved Adverse Postoperative OutcomesMedicalResearch.com Interview with Dr. Mary T. Hawn MD

Center for Surgical, Medical Acute Care Research, and Transitions,Birmingham Veterans Affairs Medical Center

University of Alabama at Birmingham, Birmingham

• Medical Research: What recommendations do you have for future research as a result of this study?

• Dr. Hawn: Further study should be done to examine other factors influencing the management of patients with cardiac stents in need of subsequent surgery and their effect on outcomes.

• Fleisher LA, Beckman JA, Brown KA, et al. ACC/AHA 2007 Guidelines on Perioperative Cardiovascular Evaluation and Care for Noncardiac Surgery: Executive Summary: A Report of the American College of Cardiology/American Heart Association Task Force on Practice Guidelines (Writing Committee to Revise the 2002 Guidelines on Perioperative Cardiovascular Evaluation for Noncardiac Surgery) Developed in Collaboration With the American Society of Echocardiography, American Society of Nuclear Cardiology, Heart Rhythm Society, Society of Cardiovascular Anesthesiologists, Society for Cardiovascular Angiography and Interventions, Society for Vascular Medicine and Biology, and Society for Vascular Surgery. J Am Coll Cardiol.Oct 23 2007;50(17):1707-1732.

• Citation:Improved Adverse Postoperative Outcomes With Revised American College of Cardiology/American Heart Association Guidelines for Patients With Cardiac StentsGraham LA1, Singletary BA1, Richman JS1, Maddox TM2, Itani KM3, Hawn MT1.JAMA Surg. 2014 Nov 1;149(11):1113-20. doi: 10.1001/jamasurg.2014.2044.

Read the rest of the interview on MedicalResearch.comContent Not Intended as Specific Medical Advice

Page 85: MedicalResearch.com:  Medical Research Exclusive Interviews November 26 2014

Full Day Preschool Increased School ReadinessMedicalResearch.com Interview with:

Dr. Arthur Reynolds PhD, ProfessorHumphrey School of Public Affairs

University of Illinois at Chicago

• Medical Research: What is the background for this study? What are the main findings?

• Dr. Reynolds: Given the high national priority on enhancing early childhood development, evidence about the relationship between full-day preschool participation and school readiness is meager.

• The study found that among about 1000 children attending 11 schools in low-income neighborhood. participation in full-day preschool at ages 3 or 4 for 7 hours per day was associated with significantly higher school readiness skills at the end of preschool in language and literacy, socio-emotional development, math, and physical health than part-day participation for 3 hours per day. This translate to about a half of a year of growth in learning. Full-day preschool was also associated with significantly higher attendance and lower rates of chronic absences. No differences were found in parent involvement in school.

Medical Research: What should patients and clinicians take away from this report?

• Dr. Reynolds: The availability of full-day preschool (prekindergarten) programs is a valuable strategy for strengthening school readiness in many domains of functioning and may therefore reduce the need for behavioral interventions to remediate or treat emergent problem behaviors that interfere with learning. The reported association with reduced chronic absences suggests that by addressing parents’ interests for a full-day center environment and increasing learning time, greater participation in early schooling is facilitated. This positive pattern of behavior can establish a strong foundation for later learning.

Read the rest of the interview on MedicalResearch.comContent Not Intended as Specific Medical Advice

Page 86: MedicalResearch.com:  Medical Research Exclusive Interviews November 26 2014

Full Day Preschool Increased School ReadinessMedicalResearch.com Interview with:

Dr. Arthur Reynolds PhD, ProfessorHumphrey School of Public Affairs

University of Illinois at Chicago

• Medical Research: What Recommendations for future research do you have a result of this work?

• Dr. Reynolds: It is important to determine the generalizability of the findings to other programs. The Midwest Child-Parent Center Program provides high quality education and family support services. The extent to which the findings are similar for different programs not having the same level of quality needs to be further examined. For example, the Midwest Child-Parent Centers has small classes run by certified BA-level teachers that receive significant support in instruction.

• The extent the gains from full-day preschool are sustained into kindergarten and later grades also needs further study. Although long-term effects of preschool programs are well-established, whether this applies to different dosages of participation (e.g., full-day vs. part-day) is much less clear.

• Citation:

• Reynolds AJ, Richardson BA, Hayakawa M, et al. Association of a Full-Day vs Part-Day Preschool Intervention With School Readiness, Attendance, and Parent Involvement. JAMA. 2014;312(20):2126-2134. doi:10.1001/jama.2014.15376.

Read the rest of the interview on MedicalResearch.comContent Not Intended as Specific Medical Advice

Page 87: MedicalResearch.com:  Medical Research Exclusive Interviews November 26 2014

Proton Pump Inhibitors May Decrease Gut Diversity, Increase C. diff RiskMedicalResearch.com Interview with:

Dr. John K. DiBaise MDGastroenterology and Hepatology

Mayo Clinic, Scottsdale Arizona

• Medical Research: What is the background for this study? What are the main findings?

• Dr. DiBaise: Despite nearly 25 years of safe and effective use of proton pump inhibitors (PPI), in recent years there have been an increasing number of reports suggesting potentially harmful effects and harmful associations with their use. One such association with PPI use has been Clostridium difficile infection (CDI) which can cause severe and recurrent episodes of diarrhea. Previous reports evaluating the microbes present within the gastrointestinal tract (ie, gut microbiome) of individuals with CDI have shown a reduction in overall microbial community diversity. We studied the gut microbiome in healthy individuals both before and after using a proton pump inhibitors for one month and found a similar reduction in microbial diversity while taking the PPI that did not entirely revert back to the ‘normal’ baseline after being off the medication for a month. While this does not demonstrate a causal association between proton pump inhibitors use and CDI, it demonstrates that PPI use creates a situation in the gut microbial environment that may increase the individual’s susceptibility to CDI.

Read the rest of the interview on MedicalResearch.comContent Not Intended as Specific Medical Advice

Page 88: MedicalResearch.com:  Medical Research Exclusive Interviews November 26 2014

Proton Pump Inhibitors May Decrease Gut Diversity, Increase C. diff RiskMedicalResearch.com Interview with:

Dr. John K. DiBaise MDGastroenterology and Hepatology

Mayo Clinic, Scottsdale Arizona

• Medical Research: What should clinicians and patients take away from your report?

• Dr. DiBaise: As with all medications, they should only be used when absolutely necessary. In the case of proton pump inhibitors they are the most effective medications to treat gastroesophageal reflux disease. They should be used at the lowest dose that provides adequate relief of symptoms and attempts to discontinue their use should be considered periodically. While this may not be appropriate in those with the most severe symptoms, less potent acid reducing drugs and non-drug anti-reflux strategies may be adequate in some individuals. It would be premature to discontinue these medications based on the results of this study. Decisions on the continued use or discontinuation of a Proton Pump Inhibitors should be made in consultation with the individual’s healthcare provider after reviewing their risks, benefits and alternatives.

Read the rest of the interview on MedicalResearch.comContent Not Intended as Specific Medical Advice

Page 89: MedicalResearch.com:  Medical Research Exclusive Interviews November 26 2014

Proton Pump Inhibitors May Decrease Gut Diversity, Increase C. diff RiskMedicalResearch.com Interview with:

Dr. John K. DiBaise MDGastroenterology and Hepatology

Mayo Clinic, Scottsdale Arizona

• Medical Research: What recommendations do you have for future research as a result of this study?

• Dr. DiBaise: This was a small study in healthy individuals – the results require further study in larger numbers of individuals, both healthy and those with chronic illness. We studied the effect of PPIs on the microbes present within the feces. Proton Pump Inhibitors are likely to have a much more profound effect on microbes present in the upper gastrointestinal tract (eg, stomach and small intestine). The effects of PPIs on the microbes present within these portions of the gut should also be studied. Finally, longer-term studies will be needed to determine whether the relationship between Proton Pump Inhibitors use and CDI is causal or not.

• Citation:

• Charlie T Seto, Patricio Jeraldo, Robert Orenstein, Nicholas Chia, John K DiBaise. Prolonged use of a proton pump inhibitor reduces microbial diversity: implications for Clostridium difficile susceptibility. Microbiome, 2014; 2 (1): 42 DOI: 10.1186/2049-2618-2-42

`

Read the rest of the interview on MedicalResearch.comContent Not Intended as Specific Medical Advice

Page 90: MedicalResearch.com:  Medical Research Exclusive Interviews November 26 2014

COPD: Nutritional Supplements Improved Outcomes and Reduced CostsMedicalResearch.com Interview with:

Refaat Hegazi, MD, PhD, MS, MPHMedical Director, Abbott Nutrition

Affiliate Research Associate Professor,The Brody School of Medicine at East Carolina University

• Medical Research: What is the background for this study? What are the main findings?

• Dr. Hegazi: This study stems from the need to address the financial and health burdens that Chronic Obstructive Pulmonary Disease (COPD) places on the United States. It is the third leading cause of death in the U.S. and costs us about $50 billion a year. It’s a devastating and chronic condition that plagues patients on a daily basis, and previous studies have shown that proper nutrition is essential for proper pulmonary function and rehabilitation.

• In a retrospective study of inpatient medical records, we found that by ensuring the nutritional needs of COPD patients were met with oral nutritional supplements (ONS), we were able to tackle the issue of cost, as well as better health outcomes. Specifically, the COPD patients that received oral nutritional supplements, experienced reduced length of hospitalization, lower average hospital costs, and lower readmission rates within 30 days, compared to those that did not.

• Medical Research: What should clinicians and patients take away from your report?

• Dr. Hegazi: I think the most important finding this study discovered is how powerful nutritional screenings and nutritional treatments are for patient recovery, as well as lowering health care costs. Relative to other medical technologies and pharmaceutical interventions, nutrition therapy is much more cost effective for hospitals and patients. We need to shift our thinking towards taking preventive measures and nutrition throughout the continuum of the disease process instead of solving issues retroactively- and that starts with avoiding malnutrition.

Read the rest of the interview on MedicalResearch.comContent Not Intended as Specific Medical Advice

Page 91: MedicalResearch.com:  Medical Research Exclusive Interviews November 26 2014

COPD: Nutritional Supplements Improved Outcomes and Reduced CostsMedicalResearch.com Interview with:

Refaat Hegazi, MD, PhD, MS, MPHMedical Director, Abbott Nutrition

Affiliate Research Associate Professor,The Brody School of Medicine at East Carolina University

• Medical Research: What recommendations do you have for future research as a result of this study?

• Dr. Hegazi: Chronic diseases, such as COPD, are not only the leading causes of death worldwide but also a main driver of health care spending. This study gives us a glimpse into how accurate nutrition diagnosis and malnutrition management can benefit chronic disease prognosis and costs. Exploring how these nutritional measures impact costs among patients with other chronic diseases, such as diabetes and heart disease, should be examined as well.

• Citation:

• Effect of hospital use of oral nutritional supplementation on length of stay, hospital cost, and 30-day readmissions among Medicare patients with COPDChest. 2014 Oct 30. doi: 10.1378/chest.14-1368. [Epub ahead of print]Thornton Snider J, Jena AB, Linthicum MT, Hegazi RA, Partridge JS, LaVallee C, Lakdawalla DN, Wischmeyer PE.

Read the rest of the interview on MedicalResearch.comContent Not Intended as Specific Medical Advice

Page 92: MedicalResearch.com:  Medical Research Exclusive Interviews November 26 2014

CPAP May Improve Depression In Patients With Sleep ApneaMedicalResearch.com Interview with: Dr. Marcus Povitz MD

Department of Community Health SciencesUniversity of Calgary, Calgary, Alberta, Canada

Adjunct Professor and Clinical FellowWestern University Department of Medicine,

• Medical Research: What is the background for this study? What are the main findings?

• Dr. Povitz: Both depression and obstructive sleep apnea are important causes of illness and have overlapping symptoms. Both feature poor quality sleep, difficulty with concentration and memory as well as daytime sleepiness or fatigue. Previous research showed that depression is common in individuals with sleep apnea, but studies investigating the effect of treating sleep apnea on depressive symptoms have had conflicting results. Our study combined the results of all randomized controlled trials of participants who were treated for sleep apnea with CPAP or mandibular advancement devices where symptoms of depression were measured both before and after treatment. We found that in studies of individuals without a lot of symptoms of depression there was still a small improvement in these symptoms after treatment with CPAP or mandibular advancement device. In 2 studies of individuals with more symptoms of depression there was a large improvement in symptoms of depression.

•Medical Research: What should clinicians and patients take away from your report?

• Dr. Povitz: Our findings indicate that treatment with CPAP or mandibular advancement devices can lead to modest improvements in depressive symptoms among people with obstructive sleep apnea. However, it is important for clinicians to realize that the studies we identified did not address the efficacy of CPAP and mandibular advancement devices compared to standard treatments for depression such as anti-depressant medications.

Read the rest of the interview on MedicalResearch.comContent Not Intended as Specific Medical Advice

Page 93: MedicalResearch.com:  Medical Research Exclusive Interviews November 26 2014

CPAP May Improve Depression In Patients With Sleep ApneaMedicalResearch.com Interview with: Dr. Marcus Povitz MD

Department of Community Health SciencesUniversity of Calgary, Calgary, Alberta, Canada

Adjunct Professor and Clinical FellowWestern University Department of Medicine,

• Medical Research: What recommendations do you have for future research as a result of this study?

• Dr. Povitz: Within in our systematic review we did not find any studies that compared treatment of OSA versus standard therapies for depression or that used clinical interviews to confirm a diagnosis of depression (the gold standard). We would like to see studies that address these limitations. Ideally, the next steps would involve a randomized controlled trial evaluating potential benefits of CPAP in patients with OSA who also have depression.

• Citation:Effect of Treatment of Obstructive Sleep Apnea on Depressive Symptoms:Systematic Review and Meta-Analysis

• Marcus Povitz equal contributor, Carmelle E. Bolo equal contributor, Steven J. Heitman, Willis H. Tsai, JianLi Wang, Matthew T. James Published: November 25, 2014DOI: 10.1371/journal.pmed.1001762

Read the rest of the interview on MedicalResearch.comContent Not Intended as Specific Medical Advice

Page 94: MedicalResearch.com:  Medical Research Exclusive Interviews November 26 2014

Inflammatory Gut Microbes May Drive Metabolic SyndromeMedicalResearch.com Interview with:

Dr. Andrew Gewirtz PhDProfessor & Associate Chair Department of Biology

Georgia State University

• Medical Research: What is the background for this study?

• Dr. Gewirtz: 2010 science paper that discovered that loss of toll-like receptor 5 altered gut microbiota to drive metabolic syndrome

• Medical Research: What are the main findings?

• Dr. Gewirtz: It is loss of tlr5 on epithelial cells that alters the microbiota to make it more pro-inflammatory that drives metabolic syndrome.

• Medical Research: What should clinicians and patients take away from your report?

Dr. Gewirtz: Failure to manage the gut microbiota can result in a an inflammatory microbiota that drives metabolic syndrome.

• Medical Research: What recommendations do you have for future research as a result of this study?

• Dr. Gewirtz: Will be importanat to develop ways to control an aberrant microbiota to make it less pro-inflammatory.

• Citation:

• Intestinal Epithelial Cell Toll-like Receptor 5 Regulates the Intestinal Microbiota to Prevent Low-Grade Inflammation and Metabolic Syndrome in MiceBy: Marie Benz MD FAADChassaing B, Ley RE, Gewirtz ATGastroenterology. 2014 Aug 27. pii: S0016-5085(14)01072-5. doi: 10.1053/j.gastro.2014.08.033. [Epub ahead of print]

Read the rest of the interview on MedicalResearch.comContent Not Intended as Specific Medical Advice

Page 95: MedicalResearch.com:  Medical Research Exclusive Interviews November 26 2014

Paid Malpractice Claims Declined Over Past DecadeMedicalResearch.com Interview with:

Michelle M. Mello, JD, PhDProfessor of Law, Stanford Law School

Professor of Health Research and Policy Stanford University School of Medicine

• Medical Research: What is the background for this study? What are the main findings?

• Dr. Mello: In this report, we examined trends in the volume and cost of medical liability claims in the US, as well as liability insurance costs, and reviewed current initiatives to reform the liability system.

• Examining publicly available data from the National Practitioner Data Bank, we found that the frequency and average cost of paid malpractice claims have been declining. The rate of paid claims against physicians decreased from 18.6 to 9.9 paid claims per 1,000 physicians between 2002 and 2013, about a 6.3% annual average decrease for MDs. Among claims that resulted in a payment, the median payment increased from $133,799 in 1994 to $218,400 in 2007, but has been declining–by 1.1% annually, on average–since 2007. In 2013 the median payment was $195,000.

• When we looked a trends in insurance premiums in several markets, using data from the Medical Liability Monitor’s Annual Rate Survey, we found greater variation from place to place. However, the overall picture was favorable. None of the locations we examined showed large increases over the last 10 years, and most showed flat or declining premiums.

Read the rest of the interview on MedicalResearch.comContent Not Intended as Specific Medical Advice

Page 96: MedicalResearch.com:  Medical Research Exclusive Interviews November 26 2014

Paid Malpractice Claims Declined Over Past DecadeMedicalResearch.com Interview with:

Michelle M. Mello, JD, PhDProfessor of Law, Stanford Law School

Professor of Health Research and Policy Stanford University School of Medicine

• Medical Research: What should clinicians and patients take away from your report?• Dr. Mello: Things have settled down in the liability environment. But that’s unlikely to last: history

teaches us that we experience regular cycles in insurance and claims costs. During this period in which costs are relatively stable, there is a window of opportunity to thoughtfully address other problems in the liability system that badly need to be fixed. These include barriers to bringing claims, discordance between the merit of a claim and whether it attracts a payment, high overhead costs, and the protracted and adversarial nature of litigation.

• In the article, we discuss several innovative approaches that have been tested in states and hospital systems over the last few years. These include communication-and-resolution programs, judge-directed negotiation, and safe harbors for adherence to evidence-based clinical practice guidelines.

• Medical Research: What recommendations do you have for future research as a result of this study?

• Dr. Mello: There is a need for further testing and evaluation of these promising approaches to improving the liability environment. Many of them are things that can be done without legislation. Some, like communication-and-resolution programs–can even be done by hospital systems and liability insurers on their own, a kind of “do-it-yourself” malpractice reform. We need to learn more about how successful these novel approaches can be in different types of settings.

• Citation:• Medical Liability Climate and Prospects for Reform

Mello MM, Studdert DM, Kachalia A. The Medical Liability Climate and Prospects for Reform. JAMA. 2014;312(20):2146-2155. doi:10.1001/jama.2014.10705.

Read the rest of the interview on MedicalResearch.comContent Not Intended as Specific Medical Advice


Recommended